Ear Reconstruction 01-22 Flashcards Preview

PRS Inservice > Ear Reconstruction 01-22 > Flashcards

Flashcards in Ear Reconstruction 01-22 Deck (101)
Loading flashcards...
1
Q

During second-stage ear reconstruction with elevation of the costal cartilage framework, a temporoparietal flap is elevated to cover a cartilage block. The blood supply of this flap most commonly comes from which of the following arteries?

A) Angular
B) Maxillary
C) Occipital
D) Posterior auricular
E) Superficial temporal

A

The correct response is Option E.

Elevation of the ear reconstruction framework can be covered by using a temporoparietal flap (TPF) if needed. The most common blood supply of the TPF is typically the superficial temporal artery but less frequently can be the posterior auricular artery or the occipital artery branches. The TPF flap can be safely raised on these less common pedicles as needed.

2
Q

A 38-year-old man has the ear deformity shown in the photograph. He would like to know if there is a name for the appearance of his ear. On the basis of the image, which of the following is the most likely diagnosis?

A) Constricted ear
B) Cryptotia
C) Microtia
D) Prominent ear
E) Stahl ear

A

The correct response is Option A.

Constricted ear deformities have a varying degree of hypoplasia of the superior third of the auricle. This patient has a mild form of a constricted ear deformity with presentation of a lop ear. Emblematic for a lop ear is the “folding” of the helical rim. In more severe cases, the antihelical fold can efface and the patient can present with a cup ear deformity. In an infant, ear molding is the treatment of choice. If the patient presents later in life, and in instances where the vertical height is near normal, excision of the folded aspect of the helix may suffice. If there is considerable lack of vertical height, the folded cartilage can be used as a flap to increase the auricular height. A variety of cartilaginous and loco-regional flap combinations have been described for this indication. The most well-known is the Banner flap.

A Stahl ear is characterized by an additional prominent fold in the scaphoid fossa, which results in an everted helical fold and pointed appearance of the ear. Like a constricted ear, this can be addressed with ear molding or with surgical correction later in life.

Prominent ears are defined as having an increased auriculocephalic angle (>25 degrees) and distance (>2 cm). This is measured between the mastoid and the posterior concha. Contributing factors can be an increased conchoscaphal angle (usually <90 degrees) due to inadequate development of the antihelical fold or an increased conchal depth (>1.5 cm). Surgery addresses one or both of those issues by excision of conchal cartilage and/or conchoscaphal sutures (Mustardé sutures).

Cryptotia is defined by the superior helical cartilage being buried underneath the temporal skin. On clinical examination, the ability to recreate a normal ear appearance with digital manipulation is the defining feature. In patients who did not have access to ear molding, a variety of techniques using locoregional flaps have been described to externalize the buried segment.

Microtia exists on a spectrum and involves absence of some or all structures of the ear (anotia). Molding is therefore not an option, and reconstructive procedures including use of autologous and synthetic material are indicated. This patient is suffering from a deformity and not an absence of auricular tissue.

3
Q

A 5-year-old girl presents to the office for surgical correction of right microtia. After discussing various treatment options, her parents decide to proceed with porous polyethylene reconstruction with temporoparietal fascial flap. Which of the following complications is most likely to occur?

A) Exposure
B) Hematoma
C) Implant fracture
D) Infection
E) Pneumothorax

A

The correct response is Option A.

The most common complication after porous polyethylene-based microtia reconstruction is exposure of the implant. In a series of over 1000 reconstructions with porous polyethylene, implant fracture was reported at 25% in the early-case series and 1.5% in the late-case series. Implant exposure was 44% and 4% in the same paired case series, respectively. Hematoma and infection are rare complications associated with implant-based reconstruction. Pneumothorax is associated with autologous reconstruction, not implant-based reconstruction.

4
Q

A 2-month-old female infant is evaluated for the ear anomaly shown in the photograph. Which of the following most accurately describes the anomaly?

A) Anotia
B) Crumple ear
C) Cryptotia
D) Lop ear
E) Stahl ear

A

The correct response is Option C.

This is cryptotia, a deformity of the ear in which the superior helical rim is buried beneath the skin of the scalp. Some studies report successful treatment with early ear molding, but this often requires elevation of the helical rim and application of a skin graft or regional flap to line the posterior aspect.

Anotia refers to complete absence of the ear structures, while lop ear is an ear that is typically constricted and has an overhang or hooding of the superior helical rim. Crumple ear is a form of constricted ear with variable cartilage abnormalities that give it an irregular appearance. Lastly, Stahl ear refers to an otherwise normal-sized ear with a third crus, usually accompanied by an evagination of the helical rim.

5
Q

A 7-year-old boy is scheduled to undergo repair of a congenital right ear deformity. On examination, the superior helix is not visible but is palpable under the scalp. There is no sulcus of the superior ear. Which of the following is the most appropriate treatment for this patient?

A) Application of scaphoconchal sutures
B) Apply ear molding for the following 3 months
C) Excision of helical skin
D) Excision of skin and cartilage from scapha
E) Incisional release with skin grafting

A

The correct response is Option E.

This patient has cryptotia, which occurs when the superior aspect of the ear is buried under the skin. The auricular cartilage can be normal under the skin. Surgical treatment involves a superior incision to release the cartilage and then resurfacing the posterior defect with local tissue or full-thickness skin grafting.

Scaphoconchal sutures, or Mustarde sutures, are used in the treatment of the prominent ear. Prominent ears typically have flattened antihelix and may have conchal bowl hypertrophy. Scaphoconchal sutures are used to recreate the antihelical fold.

Excision of helical skin is used to treat mild forms of constricted ears. Constricted ear occurs when the helical rim is deficient in circumference for the scapha, which creates a cupped shape. In mild forms, the overhanging skin of the helix can be excised. More severe forms require lengthening of the helix and scaphoconchal sutures to pull the ear back.

Ear molding is not indicated for cryptotia.

Excision of skin and cartilage from the scapha is used to treat macrotia, or large ears. The helical rim tends to be too long after the skin and cartilage is excised, therefore, a wedge of the helical rim is typically excised.

6
Q

A 5-year-old boy presents to the office for evaluation of ear deformity. Examination shows an abnormal bar of cartilage, extending from the antihelix to the helix at approximately the junction between the upper and middle thirds of the ear. Which of the following is the most likely diagnosis?

A) Constricted ear
B) Cryptotia
C) Microtia
D) Question mark ear
E) Stahl ear

A

The correct response is Option E.

In Stahl ear, there is an abnormal bar of cartilage (sometimes called the third crus), extending from the antihelix to the helix at approximately the junction between the upper and middle thirds of the ear. If that abnormal cartilage is obvious, it must be excised.

In a constricted ear, the fundamental abnormality is that the helical rim is deficient in circumference for the scapha to which it is attached. The inadequate length of the helix “constricts” the ear and forces it into a cupped shape that protrudes from the head.

In cryptotia, the superior aspect of the ear is hidden beneath the temporal scalp. In some cases, the auricular cartilage is normal and requires only to be extracted from its hiding place. Lateral traction on the ear will reveal a normal auricle.

In the question mark ear, there is excess scapha in the upper portion of the ear and a deficiency at the junction of the middle and lower thirds, resulting in a “question mark” shape.

Microtia is a congenital condition in which the cartilage of the outer ear is underdeveloped or absent.

7
Q

Which of the following otoplasty techniques is most appropriate to create the antihelical fold in patients with prominent ears?

A) Barbed resorbable suture
B) Bolster dressing with external suture
C) Furnas suture
D) Mustarde suture
E) Vertical mattress suture with resorbable suture

A

The correct response is Option D.

Otoplasty is commonly performed to correct prominent ears. Some consider prominent ears to be caused by the inadequate formation of the postauricular muscles in utero, which leads to a failure to form the antihelical fold and adequate setback of the auricle. This can result in psychosocial issues in children and adults. Since most of the ear formation occurs by age 5 years, most otoplasties are performed after that age.

Two basic concepts are associated with otoplasty: cartilage cutting and cartilage scoring. Each has its proponents and some use both techniques in reshaping the ear.

Correction of the antihelical fold is most commonly performed using the Mustarde suture. It is a non-absorbable suture placed over a weakened cartilage structure to create the antihelical crease. Stentrom described scratching the posterior cartilage to weaken the memory of the cartilage prior to application of the Mustarde suture to allow it to fold and stay folded more easily.

Furnas described the conchal setback suture to rotate the ear and decrease the angle between the concha and mastoid to approximately 25 to 35 degrees.

Vertical mattress sutures are not used for creation of the antihelical fold.

The bolster dressing with an external suture is more appropriately used after drainage of an auricular hematoma.

8
Q

An 82-year-old man is referred by his Mohs micrographic surgeon for reconstruction of a nasal defect after margins are cleared. The 8 mm × 8 mm defect is full-thickness at the alar rim and soft triangle, and involves skin, cartilage, and lining. The plastic surgeon plans to use a composite graft from the ear in a single-stage reconstruction under local anesthetic. Which of the following is the most appropriate ear donor site for reconstruction of the defect?

A) Bilobed flap for skin and anterior conchal bowl graft for cartilage and lining
B) Full-thickness harvest of tragus
C) Full-thickness helical root composite graft with cartilage limb extensions
D) Full-thickness helix wedge with primary closure
E) Posterior ear skin and concha cartilage composite graft

A

The correct response is Option C.

In this case, all three layers require reconstruction. Cartilage support and careful selection of grafts can decrease alar notching. The helical root provides three-layer reconstruction in a simple full-thickness composite graft. Three-layer composite grafts are most successful when less than 1 cm. Helical cartilage can be harvested extending beyond the skin boundaries to decrease notching and provide enhanced rim support. The attached skin on the deep surface is similar to lining and the outer skin layer is more fibro-fatty like the alae/tip skin. This donor site is rarely heavily sun damaged and can be closed primarily without significant cosmetic issues.

9
Q

An otherwise healthy 3-day-old term female infant is brought to the clinic for evaluation of a prominent ear deformity. The parents are interested in nonsurgical options, and they would like to pursue ear molding but are concerned about the complications associated with molding devices. The parents should be advised that the most common complication of such devices is which of the following?

A) Chondritis
B) Otitis media
C) Overcorrection
D) Sensorineural hearing loss
E) Skin ulceration

A

The correct response is Option E.

Although all of the answers are potential complications of treatment with ear molding, the most common complication is skin ulceration. In a recent study, skin ulceration occurred at a rate of 7.6%. Other studies have reported the rate of skin ulceration around 3%. Although chondritis and allergy to adhesives are complications, their rates were lower than 3 to 7.6%. Since this is an external system, it should not impact eustachian tube dysfunction. Ear molding does not contribute to hearing loss. Overcorrection is not likely with molding alone.

10
Q

A 2-week-old female infant is evaluated because of a prominent ear deformity. Which of the following is the most appropriate early management to address this deformity?

A) Cartilage grafting
B) External molding
C) Injection of hyaluronidase
D) Otoplasty
E) No intervention at this time

A

The correct response is Option B.

External molding is the early treatment of choice for a prominent ear deformity. Ideally, this is started by 2 weeks of age, but improvement has been seen in patients up to 3 months of age and is worth trying. Splinting is a shorter therapy used for Lop ear and Stahl ear deformation. Injectable chondroplasty using hyaluronidase is still in the trial phase, but if effective, would allow molding to be used in children older than 3 months of age. Otoplasty at 6 years of age is the alternative if molding therapy is unsuccessful.

11
Q

A 50-year-old man desires correction of his drooping right eyelid. Medical history includes traumatic injury to the eye two years ago. Physical examination shows ptosis of the right eyelid. Levator muscle function is less than 5 mm. No other abnormalities are noted. The most appropriate procedure for correction of the ptosis functions through which of the following mechanisms?

A) Activation of the frontalis muscle
B) En block removal of a portion of the posterior lamella
C) Plication of the levator aponeurosis
D) Release of muscle fibrosis
E) Repositioning of the levator aponeurosis

A

The correct response is Option A.

Only using muscle outside of the eyelid to power elevation will yield the desired action. This would involve suturing the eyelid, namely the tarsal plate, to the frontalis muscle via either alloplastic material, autogenous fascia, or a biologic product. Activation of the frontalis muscle will then elevate the eyelid independent of the levator apparatus.

In the setting of chronic non-function (two-year history of traumatic eyelid ptosis and less than 5 mm of eyelid elevation), use of local structures would not be warranted. Release, plication, or repositioning of the dysfunctional levator muscle would not be successful in restoring eyelid elevation.

12
Q

A 10-year-old boy presents with an ear injury sustained after a picture frame fell onto his head. The injury is shown in the photograph. He never lost consciousness and has no other injury. Microsurgical reattachment is not an option. Which of the following is the most appropriate initial treatment?

A) Debridement and closure
B) Dressing with petroleum gauze
C) Immediate flap reconstruction
D) Reattach as a composite graft
E) Split-thickness skin graft

A

The correct response is Option D.

While composite grafting of large ear avulsions has a globally poor outcome, the avulsed fragment in this patient is a thin piece of the helical rim that includes only a small piece of the helical rim cartilage. The shape of the defect, minimal cartilage involvement, and the fact that this was a clean injury in a young patient, makes an initial attempt to replace the tissue as a composite graft the best initial option. At worst, the tissue acts to cover the wound until a definitive reconstruction can be planned. At best, the tissue survives to some degree and salvages some of the delicate and very hard to replace helical rim contour. This patient described in the clinical scenario had 80% survival of the tissue with this technique and required no further reconstruction.

The exposed cartilage is at risk for infection and may dessicate, so a simple dressing change with petroleum is ill advised. Debridement and closure might be possible if more cartilage was removed, but this further compounds the tissue loss. A split-thickness skin graft contracts and may not take well on exposed cartilage. Immediate flap reconstruction is possible, but a flap can always be done at a later time if the composite graft does not survive.

13
Q

An 18-year-old man desires correction of a unilateral congenital Stahl ear deformity. Which of the following auricular characteristics is most likely to be observed on physical examination?

A) Accessory third crus of the antihelix
B) Conchal projection secondary to prominent mastoid
C) Enlarged conchal cartilage
D) Hyperplasia of the superior crus of the antihelix
E) Pointed thickening at the junction of the upper and middle third of the helix

A

The correct response is Option A.

Stahl congenital ear deformity can present with varying degrees of severity and is characterized by the presence of an abnormal third crus of the antihelix. There is also often an associated flattening of the helix, unfurling of the helical rim, a posterosuperior projection of the helical rim, and absence or hypoplasia of the superior crus of the antihelix.

A pointed thickening at the junction of the upper and middle third of the helix is seen in Darwin’s tubercle. Conchal projection secondary to prominent mastoid can be seen in mastoid prominence. Enlarged conchal cartilage is seen in prominent ear.

14
Q

A 25-year-old man sustained a laceration to the face from a knife in an altercation 1 year ago. He did not receive treatment at the time of injury and now has complete left unilateral facial nerve transection and paralysis. Placement of an upper eyelid gold weight is necessary for eye protection. Which of the following is the appropriate location for placement of the weight?

A) Between the tarsal plate and conjunctiva
B) Centered over the junction of the central and lateral one-third of the eyelid
C) Deep to the levator aponeurosis and superficial to the tarsal plate
D) Immediately superior to the tarsal plate
E) Superficial to the levator aponeurosis and tarsal plate

A

The correct response is Option E.

The appropriate location for placement of an upper eyelid gold weight is superficial to the levator aponeurosis and tarsal plate, with the inferior edge of the gold weight within a few millimeters of the lash line. It is placed centered over the junction of the medial and central one-thirds of the eyelid and medial limbus.

Lagopthalmos, or the inability to completely close the eyelids, is a common problem after facial nerve transection. The most common surgical treatment modality of lagopthalmos is placement of an upper eyelid gold or platinum plate to weigh down the upper eyelid, allowing for complete eyelid closure, precluding dessication and corneal damage.

15
Q

A 64-year-old woman is evaluated because of right-sided epiphora. Examination shows the condition of the patient’s eyelids is appropriate for her age. Which of the following is the best diagnostic evaluation to determine if this patient has nasolacrimal duct obstruction?

A) Goldmann tonometry
B) Jones test
C) Optical coherence tomography
D) Seidel test
E) Snap-back test

A

The correct response is Option B.

The best diagnostic test for nasolacrimal duct obstruction is the Jones test. The Jones I test evaluates lacrimal outflow under normal physiologic conditions. Fluorescein dye is instilled into the conjunctival cornice. The dye is then recovered from the nose after 5 minutes by asking the patient to blow their nose. Absence of dye from the Jones I test could mean a false-negative result, physiologic dysfunction, or anatomic obstruction. A Jones II test is performed following an unsuccessful Jones I test. For the Jones II test, the residual fluorescein is flushed from the conjunctival sac with clear saline. The investigator then asks the patient to expel the drainage from the pharynx and determines the presence or absence of fluorescein in the retrieved saline fluid. Absence of fluorescein indicates a complete nasolacrimal duct obstruction or canalicular obstruction.

The snap-back test is incorrect because it tests horizontal lower eyelid laxity. It is performed by displacing the lower eyelid inferiorly. With normal laxity, it almost immediately snaps back into place against the globe. With increased laxity, the lid will require one or more blinks to resume the normal apposition.

Optical coherence tomography is a noninvasive imaging test that uses light waves to take cross-section pictures of the retina. The Seidel test can detect leaking aqueous or exposed vitreous. Fluorescein ophthalmic strips are wet with normal saline. The fluorescein is dark orange but becomes bright green under blue light when it comes in contact with aqueous.

Goldmann tonometry measures intraocular pressure. A prism mounted on the tonometer head is placed against the cornea. When an area of 3.06 mm has been flattened, the opposing forces of corneal rigidity and the tear film are approximately equal and cancel each other out. The intraocular pressure can then be calculated.

16
Q

An 8-year-old boy is brought to the office with a congenital abnormality of the ear. A photograph is shown. When the ear is pulled on traction, the upper pole cartilage becomes visible under the skin. This abnormality is most likely caused by which of the following?

A) Anomaly of the intrinsic postauricular muscles
B) Effacement of the scaphofossa
C) Formation of a third antihelical crus
D) Hemifacial microsomia
E) Intrauterine pressure

A

The correct response is Option A.

This patient has cryptotia of the ear. It is also known as pocket ear. The upper part of the ear is adherent and the cartilage is buried under the skin in a pocket. Other deformities may be present, such as a missing upper sulcus, underdeveloped scapha, and antihelical crura.

It is caused by an anomaly of the intrinsic oblique and transverse auricular muscles.

Surgical treatment requires release from the pocket and resurfacing of the post- and retroauricular defects. In some cases otoplasty may be required for normalization. A number of techniques are described.

Intrauterine pressure may cause ear deformities, which may spontaneously resolve or are amenable to neonatal molding techniques. It does not cause cryptotia.

Stahl ear is associated with formation of a third antihelical crus.

Hemifacial microsomia is associated with microtia. Microtia is sometimes the only manifestation of hemifacial microsomia, but subtle clinical findings are often present, such as mild facial nerve weakness or soft tissue hypoplasia on the involved side.

17
Q

A 6-year-old boy with microtia is evaluated for single-staged alloplastic reconstruction. In addition to the prosthesis itself, discussing with the parents soft-tissue reconstruction should focus on tissues dependent on which of the following vasculature?

A) Facial
B) Occipital
C) Submental
D) Superficial temporal
E) Supraorbital

A

The correct response is Option D.

Although alloplastic reconstruction can be successfully performed in a single stage, 3D printed, and still allow for atresia repair, there is still a soft-tissue component that needs to be addressed. In general, the superficial temporal/temporoparietal fascia flap is the work-horse flap and is based on the superficial temporal vasculature. This requires incisions into the hair-bearing scalp; alopecia and irregular parting of the hair are possible negative outcomes. Additional skin grafting must also be discussed. The other vasculature listed is not directly important for this type of surgery.

18
Q

Which of the following best describes the main difference between the Nagata and Brent techniques of ear reconstruction for microtia?

A) Nagata is performed at 5 years of age, while Brent is only performed around 12 years of age
B) Nagata is performed in two stages, while Brent is performed in three or more
C) Nagata uses a microvascular omental flap, while Brent only uses local tissue
D) Nagata uses a porous polyethylene framework, while Brent uses autologous cartilage
E) Nagata uses local tissue for framework reconstruction, while Brent uses autologous cartilage and iliac crest bone

A

The correct response is Option B.

The primary advantage of the Nagata technique is that it typically uses fewer stages than the Brent technique. The principle difference between the two approaches lies in how the lobule and the tragus are reconstructed. In the traditional Brent sequence, lobule creation/elevation and tragal reconstruction are separate stages, while in the Nagata approach, these are created/elevated during the same stage as framework implantation.

Although microvascular techniques may be added as adjunct procedures to a particular approach, they are typically used as a salvage method for a secondary reconstruction if there is framework exposure or loss in either technique.

The Nagata and Brent techniques use autologous rib cartilage for framework reconstruction. Neither technique routinely uses iliac crest bone. The Reinisch technique is a more recently described approach to microtia ear reconstruction and uses a porous polyethylene implant, but the Brent and Nagata techniques have typically used autologous costal cartilage for the framework.

19
Q

A 66-year-old man sustains an isolated complete amputation of the right ear from an injury at work. First responders were able to recover the part, which they wrapped in a wet towel and placed on ice. An attempt is made for microvascular reanastomosis, but no vein can be identified to establish adequate outflow. Which of the following approaches will most likely yield the best long-term aesthetic outcome?

A) Dermabrasion of the epidermis of the amputated part, burial in a subcutaneous pocket, and staged elevation of the ear
B) Disposal of the amputated part with delayed costal cartilage reconstruction
C) Microvascular replantation without venous anastomosis followed by postoperative leech therapy
D) Removal of the skin of the amputated part followed by temporoparietal flap coverage
E) Replantation of the ear without microvascular anastomoses followed by postoperative hyperbaric oxygen therapy for 3 weeks

A

The correct response is Option C.

Microvascular replantation has demonstrated superiority over the other treatment options mentioned, even when venous outflow cannot be established. Indeed, a recent systematic review demonstrated no significant difference in salvage rate, transfusion rate, or postoperative appearance between cases where venous outflow was established and where it was not. Dermabrasion and subcutaneous burial (Mladick technique) was the most common method prior to the microvascular era, but has been shown to lead to inferior results compared with microsurgical approaches. The other techniques are useful as salvage procedures or in special cases where microvascular surgery is contraindicated.

20
Q

A 19-year-old woman undergoes excision of squamous cell carcinoma of the right auricle that results in a 2.5-cm defect of the mid-helix. A photograph is shown. Which of the following is the most appropriate method of reconstruction?

A) Auricular prosthesis retained by osseointegrated implants
B) Porous polyethylene implant covered by temporoparietal fascia flap and skin graft
C) Postauricular flap and conchal cartilage graft
D) Retroauricular revolving door flap
E) Wedge closure

A

The correct response is Option C.

A number of local flaps have been used to reconstruct the helix. One of the most reliable ways to reconstruct the middle third of the helix is to use a postauricular flap as described by Dieffenbach. This flap is supplied by the posterior auricular artery and vein. Wrapping the flap around a conchal cartilage graft prevents late cicatricial deformity. This flap pins the ear back and requires dividing the base of the flap in a second stage several weeks later to return the ear to a normal position. A full-thickness skin graft is used to cover the donor site defect. The Antia-Buch chondrocutaneous advancement flap may also be used for helical rim defects, but tends to result in a cupped and noticeably smaller ear for longer defects such as this.

The retroauricular “revolving door” or “flip-flop” flap is also based on postauricular skin, but it is an island flap used to reconstruct conchal bowl and occasionally antihelical defects. Wedge closure can be used for defects up to about one third of the helical rim but would result in an ear asymmetry, making the auricle noticeably smaller in this defect. A porous polyethylene implant covered by temporoparietal fascia flap and skin graft, as well as an implant retained auricular prosthesis, are options for near total and total auricular defects. They would be difficult to fixate to the remaining ear for small helical defects.

21
Q

A 7-day-old infant is brought to the office for evaluation of widened conchal-mastoid angle and an absent antihelical fold of the left ear. There is a history of maternal hepatitis and oligohydramnios. Which of the following is the most appropriate initial step in management?

A) Immediate initiation of rigid ear molding system
B) Otolaryngology consultation for inner ear evaluation
C) Reassurance that the deformity will correct itself
D) Reevaluation in 6 weeks
E) Surgical repair at 5 years of age

A

The correct response is Option A.

This infant has a “prominent/cup ear,” the most common type of ear deformity, which is characterized by a widened conchal-mastoid angle and an absent antihelical fold.

The traditional approach on ear deformities has been observation, but studies have concluded that only approximately 30% will self-correct. Ear molding techniques provide tremendous benefit to the lives of many children whose misshapen ears do not self-correct. This is possible because the higher postpartum circulating maternal estrogen will increase the amount of hyaluronic acid, a key component of the ear cartilage, and will cause a temporary malleability to the infant’s ear cartilage. Timing is important. Waiting more than 6 weeks will cause the loss of the window of opportunity to reshape the ear as the maternal estrogen is decreased. A rigid ear molding system applies a combination of anterior and posterior forces to selectively shape and expand the targeted areas (i.e., helical rim, scapha, antihelix, superior crus, concha, and lobule).

Molding in the neonatal period corrects the auricular deformities long before the onset of peer teasing and bullying. It also decreases the need for surgical correction and the associated pain and costs of surgical corrections. The results may exceed what can be achieved with the surgical alternative. Surgery is required for deformities that cannot be corrected with ear molding and is usually performed after the age of 6 years.

Since there is no inner ear pathology associated with “prominent/cup deformity,” otolaryngology consultation is not needed.

22
Q

Which of the following newborns would be the best candidate for ear molding?

A) One-day-old newborn with helical rim deformation
B) One-day-old newborn with helical rim malformation
C) One-week-old newborn with conchal deformation
D) One-week-old newborn with conchal malformation

A

Please note: Upon further review, this item was not scored as part of the examination.

The correct response is Option C.

The correct response is a patient with ear deformation at 1 week of age. When deciding on molding, the first question to ask is what is the difference between malformation and deformation. An ear malformation is when there is a partial absence of either the skin or cartilage of the external ear. These patients tend to be less optimal candidates for molding. Patients with ear deformations have fully developed but misshapen ears and therefore are better candidates for molding. The plasticity of the cartilage is due to maternal circulating estrogen which peaks at day 3 and returns to baseline at week 6, and up to 30% of infants will self correct in the first week of life. One-day-old is too soon to begin molding because in 24 hours, some of these deformities will spontaneously resolve.

23
Q

An 8-year-old boy is brought to the clinic with a right congenital ear anomaly that affects his interaction with his peers. A photograph is shown. Which of the following is the most appropriate treatment?

A) Ear molding
B) Multiple stages of surgery for rib cartilage construct, elevation of construct, and soft-tissue reconstruction
C) Surgery with cadaveric cartilage
D) Surgery with distant rib cartilage
E) Surgery with local cartilage

A

The correct response is Option E.

The photographs show Stahl ear, a congenital ear anomaly that is characterized by an abnormal third crus of the antihelix. This is a fairly uncommon deformity, and multiple surgical methods have been described. Compared with other ear anomalies, there is usually enough local cartilage in Stahl ear, so most authors recommend local cartilage flaps or grafts, and otoplasty techniques, to remove the third crus and reconstruct a more normal antihelical fold.

Ear anomalies that lack significant cartilage may require larger cartilage grafts, such as rib. Multi-stage surgery is for reconstruction of microtia, which has a severe paucity of normal cartilage and soft-tissue structures.

24
Q

A 19-year-old man sustains a partial amputation fight-bite injury to the ear. The wound has been revised and a photograph of the residual defect is shown. The patient wants the ear to have a normal appearance. Which of the following is the best option to restore normal aesthetics to the ear?

A) Addition of a silicone framework
B) Composite cartilage grafting
C) Reconstruction with a temporoparietal fascia flap with skin grafting
D) Serial fat injections
E) Staged autologous reconstruction with costal cartilage

A

The correct response is Option E.

This patient has sustained a three-dimensional and sizable defect of the ear. He is missing a portion of the helical rim, the scaphoid fossa to the antihelix. This is a difficult reconstructive challenge. The anatomic components that are missing are skin and underlying cartilage. The best option for obtaining a normal-appearing ear would be reconstruction with a cartilage framework carved to replace the cartilage and to support the overlying skin to retain the form of the ear. This could be done as a stage procedure.

Fat injection would not be adequate to replace the missing components or to give adequate shape to the ear.

A silicone framework within a scarred bed would be prone to complication, most notably infection and/or extrusion. The defect is too large to be replaced by a composite cartilage graft.

Although a temporoparietal fascia flap could potentially be used to cover a cartilage framework, used alone it would not give adequate form to reconstruct this defect.

25
Q

Parents of a healthy 3-day-old male newborn request consultation for management of their child’s bilateral lop ear deformities. Which of the following is the most appropriate next step in management?

A) Await spontaneous correction
B) Fit the infant’s head and ears for a helmet
C) Initiate ear molding devices immediately
D) Prescribe oral estrogen blocker therapy
E) Schedule bilateral otoplasty surgery at age 3 months

A

The correct response is Option C.

Newborn infant ear deformities, in order of frequency, are prominent/cup ear, lop ear, mixed deformities, Stahl ear, helical rim anomalies, conchal crus deformity, and cryptotia. Bilateral congenital ear deformities occurred in 70% of 340 patients in one study, with unilateral involvement in 30%. While surgical correction of deformed ears constituted the mainstay of therapy for decades, more recent developments in ear molding techniques have demonstrated 90% success in some studies. The presence of circulating maternal estrogen is blamed for lack of helical cartilage rigidity. This fact allows clinicians to shape the ear and, if pursued for a sufficient period of time, therapy will eliminate the deformity without need for surgical intervention. For the technique to be maximally effective, molding should be initiated by the end of the first week after birth. When begun even a few weeks later, the success rate falls to 50%. Surgical intervention is more likely to be performed just before the child reaches preschool age, allowing the ear to reach most of its adult size first. Helmets may have applicability in cranial reshaping but are not suited for management of deformed ears. Oral medication of any kind is unlikely to affect ear shape. Awaiting spontaneous correction as the child ages is the least likely means of achieving corrected ear shaping.

26
Q

A 5-year-old girl is brought to the physician because her parents are interested in correction of the unilateral ear anomaly shown in the photographs. Which of the following materials is most likely to be used in the procedure to correct this anomaly?

A) Acellular dermal matrix
B) Autogenous rib cartilage
C) Porous polyethylene
D) Silicone
E) Skin

A

The correct response is Option E.

This patient has cryptotia. The superior helical rim and scapha are not absent but lie buried under the supra-auricular skin. Although some authors have advocated expanding the superior helical framework, this is rarely indicated. In this patient with very mild auricular shortening, supplementing or reconstructing the cartilage framework with rib cartilage, polyethylene, or silicone is unnecessary. Instead, the ear framework simply released its posterior aspect and the resultant defect lined with full-thickness skin graft or any number of skin flaps raised from the postauricular region. The use of acellular dermal matrix is not an accepted method of correcting cryptotia.

27
Q

An 89-year-old man presents with a 5 x 5-cm ulcerated, biopsy-proven squamous cell carcinoma of the skin involving the pinna of the right ear. On examination, no palpable adenopathy is noted. Radiation Therapy (RT) is planned. Which of the following is the primary predictor of local control rates of RT in this patient?

A) Age of patient
B) Histology of tumor
C) Location of tumor
D) Presence of ulceration
E) Size of tumor

A

The correct response is Option E.

The National Comprehensive Cancer Network (NCCN) has published guidelines for radiation therapy for primary squamous cell carcinoma (SCC). Local control rates for SCCs less than 1 cm were 91%; those 1 to 5 cm were 76%, those larger than 5 cm were 56%.

Mohs micrographic surgery has the highest reported cure rates for SCC compared with all other modalities. However, these rates begin to drop with increased tumor size, differentiation, discontinuity, perineural invasion, and history of recurrence.

Of the head and neck sites, involvement of the ears and lips confers the highest risk for metastatic disease at 8.8% and 13.7%, respectively. In tumors greater than 2 cm in size, the risk may increase.

In the past, it was felt that lesions overlying cartilage should not be treated with radiation therapy due to the risk of chondronecrosis. It is now known that such sites can be safely treated with fractionated radiation therapy.

Radiation therapy for regional node involvement should be considered. NCCN guidelines suggest 25 (2Gy) fractions to clinically negative but “at risk” nodal basins over a 5-week period.

Age, tumor location, ulceration, and histology are secondary predictors to tumor size.

28
Q

A 28-year-old man is evaluated after sustaining a laceration across the ear from the tragus to the antitragus. A ring block of the ear using 1% lidocaine is performed prior to repair. The patient reports pain when the conchal skin near the external auditory canal is sutured. Which of the following nerves is most likely NOT anesthetized in this patient?

A) Auricular branch of the vagus (X) nerve
B) Branch of the glossopharyngeal (IX) nerve
C) Great auricular (C2, C3) nerve
D) Lesser occipital (C2, C3) nerve
E) Trigeminal (V2) nerve

A

The correct response is Option A.

The auricular branch of the vagus (X) nerve (Arnold’s nerve) innervates the external auditory canal and conchal area of the ear. This would not be blocked with a ring block. Direct infiltration of this area is needed.

The lesser occipital nerve innervates the superior pinna. A branch of the glossopharyngeal nerve innervates the middle ear. The trigeminal nerve does not innervate the ear. The great auricular nerve innervates the lobule and the majority of the pinna.

29
Q

A 7-year-old girl is evaluated because of the ear anomaly shown. CT scan of the temporal bone of the affected ear shows an absent stapes and incus. Construction using autogenous rib cartilage is planned. The girl’s family inquires about options for improving hearing to the affected ear. Which of the following is the most appropriate response?

A) Atresia repair should be performed after auricular reconstruction
B) Atresia repair should be performed before auricular reconstruction
C) A bone-anchored hearing aid should be placed after auricular reconstruction
D) A bone-anchored hearing aid should be placed before auricular reconstruction
E) Hearing cannot be improved in this patient

A

The correct response is Option C.

Staged autogenous cartilage reconstruction remains the gold standard to correct microtia. The urgency and method of treatment for associated hearing loss depends on whether the problem is unilateral or bilateral, whether external ear construction is planned, and the condition of the middle ear structures. Bilateral hearing loss can result in problems with language development and learning and requires early intervention to improve or restore hearing. This is usually done with external hearing aids in early childhood followed by atresia repair or a bone-anchored hearing aid (BAHA) later in life. Historically, most authorities have concluded that patients with congenital unilateral hearing loss naturally adjust and experience few functional implications. Consequently, most do not routinely recommend operative correction for unilateral hearing loss. There are some recent reports of improved language This examination contains test materials that are owned and copyrighted by the American Society of Plastic Surgeons. Any reproduction of these materials or any part of them, through any means, including but not limited to, copying or printing electronic files, reconstruction through memorization or dictation, and/or dissemination of these materials or any part of them is strictly prohibited. Keep printed materials in a secure location when you are not reviewing them and discard them in a secure manner, such as shredding, when you have completed the examination. Page 249 of 426 development after early biaural hearing restoration, and some clinicians now support early treatment. If correction is considered, the two primary options are atresia repair, wherein the canal is opened and the middle ear is reconstructed, or use of a hearing aid. Atresia repair is usually deferred until the external ear framework is placed or the reconstruction is complete. The success depends on the presence and normalcy of the middle ear structures. Jahrsdoerfer’s 10-point scoring system (10 being most suitable for reconstruction) grades the anatomic appearance and relationship of the middle ear structures by temporal CT scan. Middle ear reconstruction is contraindicated for a score of 5 or less. Based on the information provided, this patient with anotia (-1 point), absence of the stapes (-2 points) and absence of the incus (-2 points), has a maximum score of 5 and would not be a good candidate for middle ear reconstruction. Certainly, this procedure would not improve hearing as well as a hearing aid such as the BAHA. BAHA uses osseointegrated implants to affix a hearing aid. This provides excellent correction of conductive hearing loss and is widely used. BAHA is the best surgical option to restore hearing in this patient if so desired. However, placement should be deferred until after the autologous ear reconstruction is completed.

30
Q

A 7-year-old Asian American boy is brought to the office because of congenital cryptotia. Which of the following is the most likely pathophysiologic explanation for his condition?

A) Abnormal distribution of the intrinsic transverse and oblique auricular muscles
B) Failure of the antihelix to furl during weeks 12 to 16 of gestation
C) Failure of hillocks 3 and 4 to arise from the first and second branchial arches
D) Incomplete fusion of the six hillocks
E) Malformation of the conchal bowl

A

The correct response is Option A.

Cryptotia is a congenital ear deformity in which the upper pole appears buried beneath the mastoid skin. It is a common auricular malformation in Asians. Children with this condition often present when they are in elementary school, and are unable to wear eyeglasses. The cause of this condition is the abnormal distribution of the intrinsic auricular muscle. Malformation of the conchal bowl results in prominence of the pinna from the head. Incomplete fusion of the six hillocks does not result in cryptotia, nor does it result from failure of the two superior hillocks (3 and 4) to arise from the branchial arches. Failure of the antihelix to furl during weeks 12 to 16 results in a protruding scapha.

31
Q

A male newborn is evaluated in the hospital because of prominent ears. Nonsurgical correction with auricular molding is recommended. In order to achieve optimal correction, therapy should be initiated at which of the following ages?

A) 3 days
B) 14 days
C) 1 month
D) 6 months
E) 1 year

A

The correct response is Option A.

If treatment is initiated within the first few days of life, auricular molding can adequately and permanently treat some congenital ear deformities. Treatment must be initiated before 3 days of age and continue to 6 months of age. The efficacy of this mode of treatment is attributed to cartilage pliability due to high concentrations of circulating maternal estrogen in the first few days of life. Maternal estrogen concentrations are highest in the first 3 days of life and begin to decrease thereafter. In order to achieve good results with auricular molding, treatment must be initiated while maternal estrogen concentrations are increased and therefore cartilage pliability is high.

32
Q

Which of the following best describes the ear anomaly seen in the photograph?

A) Cryptotia
B) Microtia
C) Pixie ear
D) Prominent ear
E) Stahl ear

A

The correct response is Option A.

Cryptotia is a congenital anomaly in which the upper part of the retroauricular sulcus is absent or buried under the temporal skin. Various surgical techniques have been reported for correction of cryptotia, starting with a V-Y plasty in 1933. Conventional methods using local flap, skin grafting, tissue expander, Z-plasty, and any combined approaches correct the skin deficiency of the upper auricle. However, cosmesis can still be unsatisfying because of a visible periauricular scar, color mismatch, or a contracture deformity. Cryptotia may be treated early nonsurgically with splinting of the ear or with surgical release at a later age.

Microtia is a hypoplastic condition of the ear which includes a spectrum from complete absence of the ear (anotia) to a smaller than normal ear with normal morphology. Microtia is seen in patients with the hemifacial microsomia. In fact, patients with isolated microtia are considered to have a mild form of hemifacial microsomia.

Pixie ear deformity is a complication of rhytidectomy.

Prominent ear has a widening of the conchal-scaphal angle, an increased auriculocephalic distance, and loss of the antihelical fold.

Stahl ear, also known as Spock ear, has a third crus, a flat helix, and a malformed scaphoid fossa.

33
Q

An 8-year-old boy is scheduled to undergo otoplasty to correct prominence of the ears. Recreation of the antihelical fold, conchal setback, and lobule setback are planned. Mustardé sutures will be used to recreate the antihelical fold. The most appropriate location for placement of the Mustardé sutures in this patient is between which of the following structures?

A) Conchal cartilage and conchal cartilage
B) Conchal cartilage and mastoid fascia
C) Scapha cartilage and conchal cartilage
D) Scapha cartilage and mastoid fascia
E) Scapha cartilage and scapha cartilage

A

The correct response is Option C.

Placement of Mustardé sutures is the most common maneuver performed for a routine otoplasty and consists of horizontal mattress sutures placed between the scapha cartilage and the conchal cartilage in order to recreate the antihelical fold. Conchal setback sutures are also commonly used during an otoplasty and fixate the conchal cartilage to the mastoid fascia. Scapha cartilage would not be fixated to the mastoid fascia or else a significant pinning deformity would result. Occasionally, a wedge of excess conchal cartilage is excised to lessen the degree of conchal prominence in cases of excess. In this situation, conchal cartilage is sutured to conchal cartilage to close the resultant defect so that no contour irregularity is produced from the resection. This would not, however, have any effect on producing an antihelical fold. There is no indication for scapha cartilage to be fixated to scapha cartilage for routine otoplasty.

34
Q

A male newborn is brought to the office because of bilateral prominent ears. His ears are symmetric in size and shape. The superior crura are undefined and the conchal-scaphal angle is greater than 150 degrees. Conchal depth and projection show no abnormalities. The newborn?s parents desire correction of the ear prominence. Which of the following is the most appropriate management?

A ) Cartilage abrasion
B ) Conchal-mastoid sutures
C ) Conchal-scaphal sutures
D ) Ear molding
E ) Observation

A

The correct response is Option D.

Auricular deformation, including prominent ears, is a relatively common problem in infants. The position of the ear can change with growth and development but this process is inconsistent and unreliable. Some anomalies, such as Stahl ear, lop ear, and constricted ear, do not typically improve as the child grows. Ear molding in young infants is a very effective method to permanently improve auricular position and shape. The process is most effective in infants who are younger than 3 months of age, and is generally ineffective in older children. A custom-made mold is typically fashioned out of soft putty and affixed into the ear with surgical tape or adhesive strips. A commercial version is also available. Depending on the severity of the deformity and the age of the infant, molding is continued for several weeks to a few months. The reported results of properly done ear molding are impressive.

Cartilage abrasion along the anterior scapha and posterior cartilage suturing are surgical techniques that have been successfully employed to treat prominent ears. However, operative treatment of prominent ears is typically reserved for older children with unresolved deformities. A period of observation is unlikely to result in a significant change in ear position and will leave surgery as the family’s only remaining treatment option.

35
Q

A male newborn is evaluated because of left-sided microtia. Examination shows a small amount of vestigial cartilage and an anteriorly rotated earlobe. Which of the following is the most appropriate management?

A ) Canaloplasty prior to 1 year of age
B ) Costochondral ear reconstruction at 7 years of age
C ) Earlobe rotation after 3 months of age
D ) Excision of the vestigial ear structures prior to 1 year of age
E ) Placement of silicone framework at 7 years of age

A

The correct response is Option B.

Treatment of microtia is frequently performed with autologous tissue when the patient is approximately 6 to 7 years old. At this time, there is sufficient material at the areas of costal cartilage fusion to permit adequate reconstruction. Furthermore, normal ear development is largely complete, although the width of the ear and its distance from the scalp continue to increase until the patient is approximately 10 years old. Reconstruction is not an option for younger children, especially infants. However, surgery to place bone-conduction hearing aids will improve hearing on the affected side and may be performed when the patient is 6 to 12 months old. In patients with unilateral microtia, creation of an ear canal should be delayed until the patient is 13 to 19 years old to minimize scarring and not interfere with external reconstruction. Placement of an alloplastic framework, such as porous polyethylene, has been described in younger patients. However, silicone, which has a greater incidence of extrusion and infection, is not an option.

36
Q

A 22-year-old man comes to the office because he is dissatisfied with the appearance of his ears. Physical examination shows bilateral effacement of the antihelical folds, conchal hypertrophy, and severe prominence of the lobules. Otoplasty is performed with a posterior incision and placement of permanent sutures from the scaphal cartilage and helical sulcus cartilage to the mastoid fascia. Following surgery, the patient is satisfied with the appearance of the upper and middle parts of the ears but notes persistent prominence of the lobules. Which of the following is the most likely cause?

A ) Inadequate reduction of the antitragus
B ) Inadequate reduction of the cavum conchae
C ) Overtightening of the scapha-mastoid sutures
D ) Suturing the triangular fossa to the temporal fascia

A

The correct response is Option B.

As the cartilage angle between the cavum conchae and the antitragus becomes more acute, hypertrophy of the cavum conchae projects the lower third of the ear, forcing the lobule outward. Failure to adequately reduce the cavum conchae before suture placement will lead to persistent prominence of the lobule.

The antitragus and lobule are not directly excised during standard otoplasty. Overtightening of the scapha-mastoid sutures leads to a ‘pinned-back’ appearance. Suturing the triangular fossa to the temporal fascia corrects prominence of the upper ear.

37
Q

A 7-year-old girl is brought to the office by her parents because of prominent ears. Physical examination shows a classic cup ear deformity. Surgical correction is planned. Which of the following is the most likely complication?

A ) Epidermolysis
B ) Hematoma
C ) Hypertrophic scarring
D ) Infection
E ) Recurrence

A

The correct response is Option E.

In 1845, Dieffenbach described the first treatment of protruding ear (post-traumatic) through the resection of the posterior skin and suturing of the auricular cartilage to the mastoid region. Ely described his technique for correcting prominent ears by elective surgery in 1881. As it was then, it still is today; the most common complication is recurrence. The rate has been quoted as low as 3% to as high as 24%. Other complications include hematoma (2%), epidermolysis (3%), suture granulomas/extrusions (3%), dehiscence (1%), hypertrophic scarring (3%), keloid formation (2%), overcorrection (1.5%), infection (1%), palpability (2%), hypersensitivity (2%), asymmetry (3%), and unnatural appearance (2%).

38
Q

A 53-year-old woman comes to the office because of a 2-year history of a lump on her right ear. Physical examination shows a 2.5 × 2 × 1.5-cm neurofibroma on the concha. Resection of the skin of the entire conchal bowl and the underlying conchal cartilage is planned. Which of the following flaps is most appropriate for reconstruction of the defect?

A) Postauricular island
B) Rhomboid
C) Rim advancement
D) Rotation advancement
E) Temporalis fascia

A

The correct response is Option A.

Lesions arising on the thin anterior skin of the concha involve the perichondrium; thus, adequate excision usually requires the removal of underlying conchal cartilage. The defect may be closed with a skin graft, but closure with a local flap takes less time and provides better results in skin color, lack of contraction, and reestablishment of contour. The postauricular ?revolving door? island flap is most appropriate for repair of the defect described; it is ideally suited for particularly large defects of the concha. The larger the pedicle, the more secure the flap. Total concha replacement can occur with this skin flap. The design of the flap is partially on the posterior ear and partially on the mastoid area. A skin incision is made around the island, and the flap is raised posteriorly and anteriorly. The skin is incised through the anterior surface of the ear, and the posterior skin elevation stops at the ear mastoid groove. This vertical attachment becomes the pedicle of the flap, or the ?hinge? of the ?revolving door.? The posterior skin island can be rotated like a revolving door into the interior conchal defect, and the conchal defect is reconstructed while the posterior defect is closed primarily. This flap is also useful for smaller conchal defects. However, the pedicle will be narrower.

A rhomboid flap would be inappropriate in this area because there is not an adequate donor site for this flap. Rhomboid flaps are more appropriate in the cheek and in areas of lax skin. Rim advancement flaps and rotation advancement flaps are ideally suited for defects of the helix of the ear. Smaller defects can be adequately treated by advancing the rim, while larger defects require undermining of the postauricular skin and thus creating a rotation advancement flap. Temporalis fascia flaps are ideal for total ear resurfacing but also require skin coverage over the fascia.

39
Q

A 16-year-old boy is brought to the emergency department after sustaining an avulsion injury involving the skin of the anterior part of the left ear. Physical examination shows a 2 × 2-cm flap elevated off the conchal bowl. Which of the following arteries is most likely to be involved?

A) Infraorbital
B) Occipital
C) Posterior auricular
D) Superficial temporal
E) Transverse facial

A

The correct response is Option C.

The primary blood supply to the anterior surface of the ear is the posterior auricular artery. Arterial supply of the auricle comes from the posterior auricular artery and from the superficial temporal artery. The anterior surface of the ear is supplied by perforators of the posterior auricular artery. Only a small branch of the superficial temporal artery crosses the superior helix to supply the triangular fossa. The occipital artery provides a minor contribution to the posterior aspect of the ear.

Neither the transverse facial artery nor the infraorbital artery provides supply to the auricle. The transverse facial artery runs anteriorly to supply the parotid gland, parotid duct, and masseter muscle. The infraorbital artery provides supply to the medial angle of the orbit, the dorsal nasal area, and the upper lip.

40
Q

An otherwise healthy 2-week-old male newborn is brought to the office because his parents are concerned about the appearance of his right ear. Physical examination shows a folded upper helix. All components of the auricular structure are present. Gentle digital manipulation restores the ear to normal shape. Which of the following is the most appropriate initial management?

A) Application of a conforming splint
B) Burying of the helical cartilage in a retroauricular pocket
C) Rasping of the antihelical fold with an otoabrader
D) Resection of the superior auricular muscle
E) Surgical repair with cartilage grafts

A

The correct response is Option A.

Lop ear is among the deformations that are acquired in utero, as opposed to true congenital malformations, in which elements of the auricle may be underdeveloped or missing altogether. Circulating maternal estrogens are thought to be the cause of the softened cartilage, which lacks sufficient stiffness to support the upper helix. The cartilage should respond to shaping with a splint that is formed to match the contour of the normal helix. The splint is applied for several weeks or more. The other options involving surgical management are not indicated as primary therapy in the patient described. Resection of the superior auricular muscle may increase the superior auriculocephalic distance and aggravate the malposition of the upper helix. Cartilage rasping is indicated for the treatment of prominent ears in an older child to create a normal antihelical fold. Burying the helical cartilage beneath the mastoid skin is indicated for management of soft-tissue avulsion injury as a prelude to reconstruction. Surgical repair with cartilage grafts is unnecessary.

41
Q

A 21-year-old man comes to the office for consultation because he thinks his ears protrude. He appears self-conscious about his ears and wears his hair long to hide them. He repeatedly looks in the mirror during his evaluation. On examination, postauricular measurement shows a helical rim-to-head distance of 12 mm at the superior pole, 17 mm at the midpoint, and 23 mm at the lobule bilaterally. Which of the following is the most appropriate next step in management?

A ) Referral for psychological evaluation

B ) Re-evaluation in 1 year

C ) Resection of the conchal bowl

D ) Scoring of the anterior cartilage

E ) Conchal-mastoid suturing

A

The correct response is Option A.

The patient described exhibits traits consistent with body dysmorphic disorder (BDD). DSM-IV describes BDD as a preoccupation with an appearance that is either imagined or is a slight physical anomaly. The preoccupation must cause clinically significant distress or impairment in social, occupational, or other important areas of functioning. The degrees of emotional distress and behavioral impairment, rather than the size or nature of the physical defect, may be more accurate indicators. Although possibly more difficult to diagnose in the cosmetic surgery patient population, plastic surgeons should evaluate for possible underlying psychiatric disorders. Psychiatric illnesses are not absolute contraindications for surgery; however, cosmetic surgery is unlikely to benefit those with BDD, psychoses, or eating disorders.

Normal helical rim-to-head measurements for each third of the ear are 10 to 12 mm at the helical apex, 16 to 18 mm at the midpoint, and 20 to 22 mm at the lobule.

42
Q

A 28-year-old man comes to the office because of fever and increasing pain 1 week after partial amputation of the right ear when he was bitten during a fight. The wound was irrigated and repaired in the emergency department. Temperature is 100.4 °F (38.0 °C). Examination shows erythema and swelling of the entire ear and a small area of dehiscence containing pus along the posterior suture line. Which of the following is the most important step in management?

A ) Application of wet-to-dry dressings

B ) Completion of the amputation with sparing of cartilage in a remote subdermal pocket

C ) Intravenous antibiotic therapy

D ) Irrigation, debridement, and packing the wound open

E ) Oral antibiotic therapy

A

The correct response is Option D.

Auricular chondritis and perichondritis is a serious surgical infection requiring immediate surgical intervention as the primary course of treatment in traumatic cases. Culture swabs should also be obtained and will guide antibiotic therapy for associated cellulitis; however, broad spectrum coverage initially is appropriate. Chondritis complicating elective otoplasty is sometimes handled in a more stepwise approach, sometimes initiating intravenous antibiotics while removing a few sutures to allow drainage and/or insertion of a small irrigating catheter. Without a reasonable response in these cases, then standard open irrigation and debridement are done with removal of all sutures, and repeat otoplasty correction is deferred until several months after resolution of the infection.

Antibiotics alone, orally or intravenously, for localized suppurative chondritis are likely to be ineffective without surgical treatment. Likewise, topical dressings without formal opening of the suture line for wide exposure and drainage of the infected cartilage would also be ineffective.

Completing the amputation when the tissues still appear viable is overaggressive at this stage. If and when the majority of the tissue appears unable to sustain sufficient circulation to support its viability, then discarding precious tissue is justified. Salvaging the cartilage framework component in a remote subdermal pocket may be useful; however, the cartilage is infected in this scenario and would require caution, including thorough debridement and lavage with antibiotic solution and close monitoring of the bank.

43
Q

An 8-year-old girl (shown) is brought to the office because of bilateral microtia. She has severe conductive hearing loss bilaterally. Her middle ear ossicles are fused, and she is not a good candidate for middle ear reconstruction. Reconstruction of the ears using autologous rib cartilage and placement of bone-anchored hearing aids (BAHAs) are planned. Which of the following is the most appropriate time for placement of the BAHAs?

A ) During elevation of the carved-rib framework

B ) During placement of the carved-rib framework

C ) During rotation of the earlobe

D ) Following completion of the ear reconstruction

E ) Prior to the ear reconstruction

A

The correct response is Option D.

Patients with severe microtia almost invariably have conductive hearing loss resulting from abnormalities of the middle ear. In some patients, the auditory ossicles are sufficiently formed to allow attempted reconstruction of the auditory canal and the middle ear ossicles. Eligibility for this procedure depends upon the development and shape of the auditory ossicles (Jahrsdoerfer grade), as determined by a specialized CT scan of the temporal region. In patients with unilateral microtia and normal contralateral hearing, assistive devices or middle ear reconstruction are

rarely indicated and would serve only to improve sound localization. In these patients, protection of hearing in the normal ear is paramount. When bilateral conductive hearing loss is present, methods to improve hearing competence include external hearing aids, bone-anchored hearing aids (BAHAs), and middle ear reconstruction in selected patients. The BAHA is affixed in the mastoid region using an osseointegrated implant. Since this can compromise the integrity and mobility of the skin envelope that will cover the autologous rib cartilage framework, it is often recommended that placement of the BAHA device be deferred until after the ear construction is complete.

44
Q

A 6-year-old boy is undergoing the first stage of total auricular reconstruction for correction of microtia. A finely detailed rib cartilage construct is carved. Which of the following aspects of this patient €™s postoperative care should be most effective in safely promoting adherence of the new cartilage construct to the skin cover?

A ) Application of cold compresses

B ) Application of pressure dressings

C ) Closed suction drainage

D ) Conforming splinting

E ) Open drainage incisions

A

The correct response is Option C.

Attentive postoperative management is imperative in total auricular reconstruction. Skin coaptation to the carved cartilage construct is best provided by continuous closed suction drainage for the first five postoperative days. The reported complication rate is less than 1%.

The quantity and quality of drainage can be monitored, and any potential hematoma is removed before it obscures the framework details. This can also be used to aid flap coaptation in cases in which temporoparietal or omental flap coverage is needed. Cold compresses are not effective in promoting adherence of the skin flap to the underlying framework.

Pressure dressings risk skin necrosis over the new cartilage framework and are not preferred over closed suction drainage. One author experienced a 33% skin necrosis complication rate in the first 15 cases in which bolster sutures were used to secure pressure bandages. In some techniques, carefully placed bolster sutures have been advocated to prevent hematoma, although the risk of skin necrosis remains. Additional skin incisions may adversely affect the final result and may also compromise the skin flap.

45
Q

A 4-year-old girl is brought to the office because her parents are concerned about an abnormality of her ear (shown). Which of the following is the most likely diagnosis?

A ) Cup ear
B ) Macrotia
C ) Microtia
D ) Prominent ear
E ) Stahl ear

A

The correct response is Option E.

Stahl syndrome is a hereditary auricular deformity caused by an abnormal cartilaginous pleat, which extends from the crus antihelix to the edge of the helix, deforming the regular curvature of the ear. The prominent ear is typically comprised of (1) an increased conchoscaphal angle, (2) deepened conchal bowl, and (3) prominent lobule. Microtia refers to a spectrum of congenital external ear deformities ranging from a slightly smaller ear to almost complete absence.

46
Q

A 27-year-old man is brought to the emergency department two hours after sustaining a laceration of the left ear in a fight. Physical examination shows a 3-cm laceration that extends from the helix to the conchal bowl. A ring block is performed. The wound is cleaned and suturing is begun. The patient grimaces and withdraws during suturing of the section of the laceration that extends to the concha. The most likely cause is inadequate anesthesia of which of the following nerves?

A ) Auricular branch of the vagus (X)

B ) Auriculotemporal

C ) Great auricular

D ) Lesser auricular

E ) Lesser occipital

A

The correct response is Option A.

The majority of the external ear can be readily anesthetized by placing a ring of local anesthetic around the ear. The exception is the concha and the posterior auditory canal. These structures are innervated by the Arnold nerve, the auricular branch of the vagus nerve (X), which travels along the ear canal. This nerve, and therefore the concha and the posterior auditory canal, is not successfully anesthetized with a ring block of the ear. If anesthesia is needed in this area, local infiltration is required.

47
Q

Microtia is an auricular deformity most commonly found in patients with which of the following syndromes?

A ) Apert

B ) Binder

C ) Crouzon

D ) Pfeiffer

E ) Treacher Collins

A

The correct response is Option E.

Treacher Collins syndrome is characterized by bilateral abnormalities of structures within the first and second branchial arches. Treacher Collins syndrome has anomalies in structures derived from the first branchial arch, groove, and pouch. It is considered to be autosomal dominant in its inheritance with variable expressivity. Features include a convex facial profile with a retrusive lower jaw and chin, down-slant of the palpebral fissures, lower eyelid colobomas, partial absence of eyelid cilia, absent or malformed external ears, hypoplasia of the malar bones, and variable cleft palate.

Binder syndrome is characterized by nasomaxillary hypoplasia. Apert, Crouzon, and Pfeiffer syndromes are all craniosynostotic syndromes that result in mid-facial hypoplasia and a concave facial profile of varying degrees. These syndromes do not typically present with microtia.

48
Q

A 5-year-old boy is brought to the office by his parents for consultation regarding correction of Stahl’s ear deformity. Which of the following is the most appropriate surgical management?

(A) Partial detachment of the folded segment of helical cartilage from the scapha and repositioning of the helix

(B) Placement of sutures from the conchal bowl to the mastoid fascia

(C) Rasping of the anterior surface of the antihelical cartilage to create the antihelix

(D) Release of the superior auricle from the temporal skin with skin grafting to create a retroauricular sulcus

(E) Wedge excision of the third crus with helical advancement

A

The correct response is Option E.

Stahl’s ear is an auricular deformity that primarily includes a third antihelical crus. One of the many methods of repair involves wedge excision of the third crus with helical advancement. The other techniques listed are used for other deformities of the ear.

The constricted ear is a deformity that involves a €œconstricted € helical rim with the superior portion of the helix often folding over the scapha. This deformity has also been referred to as €œcup € or €œlop € ear and occurs with varying degrees of severity. One method of repair includes partially detaching the helix from the scapha and resuturing it to the scapha at the proper angle.

Prominent ears are characterized by a wide conchoscaphal angle and are often associated with flattening of the antihelical fold. Various corrective methods have been described, including rasping of the anterior surface of the antihelix, placement of retention sutures to recreate the antihelical fold, conchal bowl resection, and suturing of the conchal bowl to the mastoid fascia.

Cryptotia is an adherence of the superior portion of the helix to the temporal skin with varying degrees of severity. Surgical correction involves release of this abnormal connection and frequently requires skin grafting, depending on the severity of the deformity.

49
Q

A healthy 7-year-old boy with anotia undergoes reconstruction of the right ear via implantation of a high-density porous polyethylene prosthesis. Compared with use of autologous cartilage, which of the following is the most likely result of this procedure?

(A) Higher incidence of contour deformities

(B) Higher incidence of extrusion

(C) Increased likelihood of malposition

(D) Lower incidence of infection

(E) More resorption of the implant over time

A

The correct response is Option B.

High €‘density porous polyethylene (Medpor) is an alloplastic implant material that is nonresorbable and highly biocompatible. It has an intramaterial porosity with a pore size between 125 and 250 μm, which permits extensive fibrovascular ingrowth throughout the implant. Although limited bony ingrowth may occur in select clinical circumstances, the material should not be considered truly osteoconductive. This bony and fibrovascular ingrowth can make this type of material somewhat difficult to remove. Studies have shown less underlying bone resorption occurs with high €‘density polyethylene than with other implant materials. As an alloplastic material, it should not be placed in contaminated wound beds, because it can get easily infected, and should always be positioned in areas of adequate soft tissue cover to prevent extrusion. The incidence of extrusion is higher in microtia reconstruction and nasal dorsum augmentation for this reason.

50
Q

A 17 year old boy is brought to the office three days after he sustained trauma to the ear during practice with his wrestling team. A photograph is shown. Which of the following is the most appropriate management?

(A) Incision and evacuation followed by application of a bolster dressing

(B) Intravenous administration of an antibiotic for 48 hours followed by oral administration of an antibiotic for five days

(C) Needle aspiration

(D) Observation for two weeks followed by drainage if fluid is present in the wound

(E) Resection of the posterior cartilage and sculpting of the anterior cartilage

A

The correct response is Option A.

The history and photograph of the patient described indicate an othematoma caused by shearing of the skin from the cartilage. Early diagnosis of this hematoma will prevent cauliflower ear deformity. Complete evacuation via a small incision with application of a bolster dressing to prevent reaccumulation of the fluid or blood is the most appropriate management.

The use of intravenous antibiotics to treat the hematoma is inappropriate, and there is no sign of infection. Needle aspiration may help, but the hematoma is unlikely to be completely evacuated. Allowing the hematoma to remain can result in calcification. Observation may lead to cauliflower ear deformity. Resection of the posterior cartilage and sculpting of the anterior cartilage is the treatment option for chronic calcification or cauliflower ear deformity, but not for acute othematoma.

51
Q

A 55 year old woman is referred to the office by her primary physician because she has had a painful, chronic area of scabbing on the right ear for the past three months. Physical examination shows a 2 x 1-mm area of skin ulceration on the superior helix of the ear. Surrounding inflammation and exposed cartilage are noted. Which of the following is the most appropriate initial step in management?

(A) Daily wound dressing with bacitracin ointment and avoidance of pressure on the ear

(B) Excision with 5-mm margins and coverage with a retroauricular trap door flap

(C) Excision with 5-mm margins and reconstruction with an Antia €‘Buch advancement flap

(D) Excisional biopsy and primary wound closure

(E) Injection of triamcinolone and daily topical application of hydrocortisone cream

A

The correct response is Option D.

The patient described most likely has a benign condition referred to as chondrodermatitis nodularis helicis, an inflammation of the cartilage of the ear that often leads to a painful open area. This condition can mimic a skin cancer on physical examination. Unless the area is clearly benign, biopsy is warranted to rule out skin cancer. Recurrence rate for this condition is high. Excisional biopsy and primary wound closure will give a pathological confirmation and may suffice as treatment.

Daily wound dressing with bacitracin ointment or injection of triamcinolone and daily topical application of hydrocortisone cream are both inappropriate because neither would give the appropriate tissue diagnosis. Excision and reconstruction or coverage with a flap are unwarranted given no pathological diagnosis.

Chondrodermatitis nodularis helicis is a condition of unknown etiology predominantly found in older men and often is associated with trauma from sleeping. It begins as an area of cartilage inflammation and then ulcerates through the skin. The treatment is excision of the cartilage and closure of the skin. Recurrence rates remain high. Preventive measures to decrease the recurrence rate include avoidance of sleeping on the affected ear.

52
Q

A 2-week-old female infant is evaluated because of a prominent ear deformity. Which of the following is the most appropriate early management to address this deformity?

A) Cartilage grafting
B) External molding
C) Injection of hyaluronidase
D) Otoplasty
E) No intervention at this time

A

The correct response is Option B.

External molding is the early treatment of choice for a prominent ear deformity. Ideally, this is started by 2 weeks of age, but improvement has been seen in patients up to 3 months of age and is worth trying. Splinting is a shorter therapy used for Lop ear and Stahl ear deformation. Injectable chondroplasty using hyaluronidase is still in the trial phase, but if effective, would allow molding to be used in children older than 3 months of age. Otoplasty at 6 years of age is the alternative if molding therapy is unsuccessful.

53
Q

A 57-year-old man comes to the office for follow-up examination eight days after he underwent composite grafting from the right auricle for repair of a 1.4-cm alar defect. He does not smoke cigarettes. Physical examination shows a 5-mm region of yellow eschar in the center of the graft, which is cyanotic. Which of the following is the most appropriate next step in management?

(A) Application of ice compresses

(B) Debridement of eschar and coverage with a local flap

(C) Leech therapy

(D) Continued observation

A

The correct response is Option D.

Auricular composite grafts are frequently used to reconstruct small defects of the alar rim. Unlike skin grafts, composite grafts only interface with the recipient bed along their perimeter; therefore, graft size should be no larger than 1.5 cm in diameter, so the center of the graft is never more than 5 €“8 mm from a blood supply.

The clinical appearance of a healing composite graft follows a reproducible pattern. The graft can look very white for the first six to eight hours. A slight pink tinge thereafter signals the earliest stage of revascularization. Within one to two days, the graft should appear cyanotic secondary to venous congestion, and this can persist for approximately one week. The graft will ultimately become pink and then more red as revascularization intensifies. The redness will fade, and the tissue acquires its final tone within six months to one year.

A lingering whitish area in the center of a cyanotic graft most likely represents impending necrosis. However, even if eschar develops, it should not be debrided. It may only be superficial, with the underlying dermis and cartilage still viable. Removing the eschar may dislodge the surviving portion of the graft and also places it at increased risk for infection. The eschar will likely slough within three to four weeks. Therefore, continued observation is the most prudent initial step in management.

Postoperative cooling is recommended to decrease the metabolic rate of the grafted tissue until secondary revascularization has taken place. This therapy would likely be ineffective one week after surgery and may compromise the neovascularization through vasoconstriction.

54
Q

A 24-year-old man who sustained total amputation of the left ear during an explosion three months ago comes to the office for consultation regarding reconstruction of the ear. Physical examination shows loss of the auricle and tragus with scarring of the skin and scalp surrounding the external auditory canal. Which of the following materials is most appropriate for the first stage of the reconstruction of this patient €™s ear?

(A) Full-thickness skin grafting to replace the scar and subsequent expansion

(B) Radial forearm flap and Silastic framework

(C) Temporalis muscle turnover flap and rib framework

(D) Temporoparietal flap and rib graft framework

(E) Tissue expansion of retroauricular skin and Medpor framework

A

The correct response is Option D.

The goal of reconstruction of the ear is to restore the cosmesis of the auricle and the function of the superior helical rim in providing support for eyeglasses. Total reconstruction of the ear most likely requires two to three stages. The first stage typically involves creating a cartilage framework and placing it under vascularized tissue. In the scenario described, an ipsilateral temporoparietal fascia pedicled flap or a contralateral free temporoparietal fascia flap (TPFF) covered with a split-thickness skin graft (STSG) is most appropriate. STSGs are generally used because they will shrink and form fit to the auricular cartilage over the TPFF or in the posterior auricular skin crease.

Other options include a local tissue flap or a pre €‘expanded local tissue flap. A tissue expander can be placed and may be appropriate for some patients. However, it is not likely to be successful in the patient described because of the scarred skin in the region of expander placement. A second surgery is usually necessary to recreate the posterior auricular crease and the tragus or lobule. In young patients, rib cartilage is the most common first choice for autogenous framework creation because it has less risk of exposure and is readily available. In the patient described, Medpor would not be the most appropriate choice for the first stage of reconstruction because it is associated with a high exposure rate when used with thin atrophic or scarred soft tissue. The quality and quantity of skin redraping the auricular framework play a significant role in the aesthetics of the final result. In older patients with postablative defects and compromised surrounding tissue, prosthetic reconstruction can be offered as a viable, perhaps simplistic, solution to staged auricular reconstruction. A younger patient is seldom compliant with a prosthesis, and autologous reconstruction is a better option.

55
Q

A 16 €‘year €‘old boy who underwent costochondral reconstruction of the right ear 20 days ago because of microtia has had pain, erythema, fluctuance, and swelling of the ear as well as fever for the past 48 hours. A photograph is shown. In addition to intravenous administration of an antibiotic, which of the following is the most appropriate management?

(A) Application of mafenide acetate (Sulfamylon)

(B) Incision and drainage, washout, and placement of irrigating drains

(C) Incision and removal of the cartilage construct and placement of irrigating drains

(D) Needle aspiration and application of a compressive bolster dressing

(E) Observation

A

The correct response is Option B.

The clinical scenario described is dire because of the likely loss of the costochondral ear reconstruction. The cartilage may have gained enough vascularity to help avoid loss of the construct. In addition to intravenous administration of an antibiotic, incision and removal of the construct must be considered, but a trial of incision and drainage, washout, and placement of irrigating drains is warranted in an attempt to avoid loss of this graft. If the fever and cellulitis do not improve quickly, then removal of the construct is warranted as a secondary procedure.

Application of Sulfamylon, needle aspiration, and observation are not aggressive enough for this severe infection.

56
Q

A 50-year-old man is brought to the emergency department 90 minutes after he sustained injuries in a motor vehicle collision. Physical examination shows a severe abrasion injury to the right ear with partial degloving of the superior helical rim extending into the concha. The skin over the upper third of the ear is missing at variable depths. Which of the following is the most appropriate initial step in management?

(A) Conservative debridement and topical wound care

(B) Debridement and coverage with a local flap

(C) Debridement and skin grafting

(D) Debridement, coverage with a temporoparietal fascia flap, and skin grafting

(E) Resection and primary wound closure

A

The correct response is Option A.

The patient described has a partial-thickness injury involving the upper third of the ear. It is often not clear initially if the perichondrium is viable in this situation. Although the patient may require coverage of the exposed cartilage with a flap, initial management is most often conservative. The most appropriate initial step is conservative debridement and topical wound care including dressing changes until the devitalized tissues demarcate. This management allows for a full assessment of the extent of the injury. It is important that the dressings prevent desiccation and infection of the exposed perichondrium.

Debridement followed by immediate skin grafting is not appropriate because skin graft take is often unreliable if the cartilage is exposed. Also, the extent of tissue injury caused by shearing and blunt trauma is difficult to assess initially because of the soiled and edematous tissues of the fresh trauma.

Debridement, coverage with a temporoparietal fascia flap, and skin grafting is effective in providing a bed for skin grafting but is not the most appropriate initial step in management.

Resection and closure of a defect of this size can be performed, often with an Antia €‘Buch flap after wedge resection with a pentagonal design. However, this procedure will reduce the size of the ear and, therefore, is not appropriate.

57
Q

A 28-year-old man comes to the office six months after he sustained partial amputation of the left ear due to a human bite (shown - defect in red). For successful reconstruction, which of the following anatomic areas of the ear must be addressed?

(A) Antihelix and triangular fossa

(B) Antitragus and intertragic notch

(C) Concha and scaphoid fossa

(D) Crus of helix and tubercle

(E) Helix and lobule

A

The correct response is Option E.

Even though the upper helix is still present, the middle helix below the auricular tubercle and the lower helix in continuity with the lobule are absent. Precise understanding of the anatomy of the ear is important in the planning of reconstruction regarding framework and soft-tissue requirements.

The antihelix is intact and represents the border of the defect. Inferiorly, the antihelix ends in the antitragus; superiorly, it divides into two crura. The triangular fossa separates the superior crus from the inferior crus. The intertragic notch is located between the tragus and antitragus. The concha constitutes the bowl €‘shaped area in the center of the ear from which the root or crus of the helix arises. The scaphoid fossa is the groove between the upper helix and the superior crus of the antihelix. The auricular tubercle is a thickening in the helix located between the upper and middle thirds of the helix.

58
Q

A 22-year-old woman sustains an injury to the middle third of the external ear when she is bitten by a dog. She is concerned about her appearance because she works as a model. Physical examination shows a full-thickness defect (2.5 x 2 cm) involving the rim, antihelical fold, and concha. Which of the following techniques for reconstruction of the ear will best maintain symmetry of the ears?
(A) Postauricular transposition skin flap
(B) Postauricular €œrevolving door € island flap
(C) Rim advancement
(D) Triangular kite flap
(E) Wedge resection with direct closure

A

The correct response is Option A.

In the older patient, a partial middle ear defect can be treated by wedge resection and direct closure. In a younger patient, the cupping of the ear resulting from this approach would be aesthetically unacceptable; therefore, a more complete reconstruction is required. A full-thickness loss of rim, antihelical fold, and a variable amount of concha can be reconstructed only with a flap.

The postauricular transposition skin flap is the best solution. This flap is based at the edge of the hairline, and the width of the flap is equal to that of the defect. After 10 days, the base of the flap is divided and the remainder of the flap is used to resurface the posterior part of the ear. The donor area, if small, will heal spontaneously or a skin graft may be used.

A rim defect can be treated with a small triangular kite flap or a rim advancement flap. The postauricular Arevolving door@ island flap is ideal for defects of the conchal area.

59
Q

A 2-month-old boy is brought to the office by his parents for consultation regarding congenital lop ear. On the basis of the physical examination, nonoperative correction of the deformity with thermoplastic splinting is planned. The parents ask how long their son will be required to wear the splint. The physician tells the parents that if their son wears the splint as directed, the most likely length of time between application of the device to full correction of the deformity is which of the following?
(A) One week
(B) Two months
(C) Four months
(D) Eight months
(E) One year

A

The correct response is Option B.

Nonsurgical treatment of various congenital auricular deformities has been reported in children of neonatal age. More recently, this has also been achieved in children who are several years of age. The splints are made from a malleable thermoplastic material and are applied to the ear by hand until a normal form is attained. Remodeling can be performed once per week until a desired correction is achieved. In one study involving 290 patients, 70% of cases could be treated with good results. The average time for treatment was 1.9 months (range, one week to four months). All patients showed remarkable improvement within the first one to two weeks after beginning the treatment. Young age, cooperation of the parents and patient, elasticity of the cartilage, and type of deformity are all contributing factors in the time required for treatment.

60
Q

A 7-year-old boy is brought to the office by his parents for consultation regarding prominence of the entire ears. On physical examination, the antihelical folds are intact. On the basis of this finding, which of the following deformities is the most likely cause of prominence of the ears in this patient?
(A) Hypertrophy of the concha
(B) Hypertrophy of the helix
(C) Hypertrophy of the tragus
(D) Lobular protrusion
(E) Prominent antitragus

A

The correct response is Option A.

Conchal hypertrophy is the second most common deformity and is usually bilateral. The most common deformity is loss of the antihelical fold. The ear deformities can be classified as: 1) absent or insufficient antihelical folding, 2) conchal hypertrophy, 3) both absent or insufficient antihelical folding and conchal hypertrophy, 4) any of the aforementioned deformities with lobular protrusion. If the patient is also lacking an antihelical fold, a large concha can be reduced by placing the antihelical fold more medially in the concha, thus reducing its height. If this is not enough, plication of the concha to the fascia can be performed with permanent sutures. In cases in which further correction is needed, a posterior wedge can be taken out of the conchal cartilage.

61
Q

A 22-year-old man who is a professional boxer comes to the emergency department because he has a hematoma on the anterior surface of the left ear one hour after he sustained a direct blow to the ear during a match. Which of the following is the most appropriate management?
(A) Application of pressure and ice to the ear
(B) Application of a mold to the ear
(C) Percutaneous drainage of the hematoma
(D) Open drainage of the hematoma and application of a bolster dressing
(E) Open drainage of the hematoma and reshaping of the cartilage

A

The correct response is Option D.

Auricular hematomas are common in athletes, such as wrestlers and boxers, in whom direct trauma to the ear is possible. Untreated subperichondrial hematomas can lead to formation of new cartilage and the appearance of a thickened and deformed ear (cauliflower ear). To maintain the natural contour of the ear, appropriate management consists of open drainage of the hematoma followed by closure of the incision and application of a tie-over bolster dressing. Percutaneous drainage techniques or simple cold compresses are generally not adequate in completely removing the hematoma. Molds can be used to shape the ear of an infant but not of an adult. Cartilage reshaping techniques, such as scoring, are not necessary in the management of acute auricular hematomas but may be needed for later reconstruction of cauliflower ear.

62
Q

A 70-year-old man who underwent total resection of the right ear followed by radiation because of squamous cell carcinoma one year ago comes to the office for consultation regarding reconstruction of the ear. He says he needs the ear to support his eyeglasses. Medical history shows chronic obstructive pulmonary disease. Which of the following reconstruction techniques is most appropriate in this patient?
(A) Fixation of autogenous cartilage under local flaps
(B) Implantation of osseointegrated auricular prostheses
(C) Placement of porous polyethylene framework and coverage with a radial forearm free flap
(D) Placement of silicone rubber (Silastic) framework and coverage with a temporoparietal flap
(E) Tissue expansion and placement of porous polyethylene framework

A

The correct response is Option B.

Ear reconstruction techniques are needed to address a variety of congenital, traumatic, and ablative defects. Both an intricate framework and stable covering are needed to obtain a consistent and aesthetically pleasing reconstruction. Cartilage has been the framework of choice for many years, but this involves some donor morbidity. Silastic and porous polyethylene frameworks offer synthetic alternatives but can become infected or exposed. Covering can be accomplished by local tissue if available, expanded tissue, and local or distant skin flaps.

Prosthetic ears have been available for many years but were limited by the harsh adhesives necessary to keep them in place. The use of osseointegrated titanium implants for the retention of dental prostheses opened a new possibility for implant fixation in other areas of the body.

Several recent studies document the recommended indications for autogenous versus prosthetic techniques. One study of 98 patients recommends autogenous reconstruction for pediatric microtia patients and prosthetic reconstruction for traumatic or ablative defects in adults. In addition, prosthetic ears are also recommended for severe soft tissue hypoplasia, low or unfavorable hairline, and failed autogenous reconstruction. Another study of 55 patients recommended autogenous reconstruction for classic microtia, relatively normal lower one third of the ear, patient preference, and noncompliant patients. The principal indications for osseointegrated implants were major cancer extirpation, poor local tissue, absence of the lower half of the ear, salvage following unsuccessful surgery, and poor operative risk patients.

In this case of an older patient with medical problems and poor local tissue, the fastest and safest option is a prosthetic ear implant. His radiation would preclude tissue expansion or a temporoparietal flap. A rib donor site would have significant morbidity in a patient with chronic obstructive pulmonary disease. A free flap is also high risk in this patient, with its longer anesthetic time and need for anticoagulation.

63
Q

A 25-year-old man has congestion of the right ear 10 hours after he underwent replantation of the auricle due to near-total amputation of the ear in a motor vehicle collision. During microscopic surgery, a single small artery was anastomosed to relocated superficial temporal vessels. No vein could be located. Which of the following is the most appropriate management of this patient’s current symptom?
(A) Elevation of the head
(B) Topical application of nitroglycerin
(C) Leech therapy
(D) Vein grafting
(E) Surgical reexploration

A

The correct response is Option C.

Microsurgical ear replantation provides another option in select cases of ear amputation, providing the potential for an unsurpassed aesthetic result. In those relatively few cases in which it is attempted, the small size of the vessel and the component of avulsion can make primary repair of the vessels difficult. Venous congestion occurs to some extent in nearly every case and is the most common cause of postreplantation complications. There have been several case reports of successful ear replantation without a venous anastomosis, although venous anastomosis is recommended whenever possible. Despite this, postoperative congestion can be managed with leech therapy for vascular compromise. Arterial compromise demands reexploration, whereas venous compromise can be managed nonoperatively. Topical vasodilators, such as nitroglycerin, are not recommended for venous congestion. Efforts should be made to improve outflow; therefore, conservative management is not recommended. Elevating the patient=s head may be useful but does not replace the proven effectiveness of leeches and heparin in clear cases of congestion.

64
Q

A 2-year-old boy with isolated unilateral microtia is brought to the office by his parents for consultation regarding reconstruction of the ear. Which of the following is the most appropriate management?
(A) Delay of reconstruction of the ear until 6 years of age
(B) Fitting of a prosthetic ear
(C) Placement of a postauricular tissue expander
(D) First-stage reconstruction of the ear using a cartilage rib graft
(E) First-stage reconstruction of the ear using a polyethylene prosthesis

A

The correct response is Option A.

Reconstruction of the ears should be delayed until at least 5 to 6 years of age and preferably until 7 to 8 years of age. At younger ages, patients often do not strongly desire the reconstruction, are more likely to have difficulty with postoperative compliance, and may not have rib cartilage large enough to create an auricular framework if autogenous cartilage is to be used. In addition, reconstructing an ear too early in life may result in an ear that is too small if it does not grow with the patient.

Tissue expansion of postauricular skin may be used during ear reconstruction but should be delayed until at least 5 to 6 years of age. A criticism of tissue expansion for ear reconstruction is that the expanded skin is less supple than unexpanded skin and is less able to conform to the anatomic details of an autologous or prosthetic ear cartilage framework.

Prosthetic ears are not appropriate for young children because they are usually too physically active to keep a prosthesis attached and are too young to care for a prosthesis.

First-stage ear reconstruction can be successfully accomplished using autologous cartilage or prosthetic material to recreate the underlying ear framework but should be delayed until this child is older.

65
Q

A 14-year-old boy has the bilateral congenital ear deformity shown above. In correction of this deformity, alteration of which of the following anatomic areas is most appropriate?

(A) Antihelix and helical rim
(B) Concha and antihelix
(C) Concha and lobule
(D) Helical rim and lobule
(E) Lobule and scapha

A

The correct response is Option B.

The external ear consists of a three-tiered cartilaginous framework with skin on its anterior surface. The complete ear includes the scapha, concha, helix, antihelix, tragus, and lobule as shown below.

In most cases, prominent ears result from inadequate formation of the antihelical fold. In many cases, conchal hypertrophy contributes to or is the primary cause of the prominence. To address this patient’s prominent (protruding) ears, conchal reduction must be performed via an anterior or posterior approach and an antihelical fold must be created. The anterior approach allows reduction of the skin and cartilage of the concha; the posterior approach allows only cartilage reduction.

66
Q

The antihelix and antitragus of the external ear arise from which of the following embryologic structures?

(A) First branchial arch
(B) First branchial groove
(C) First pharyngeal pouch
(D) Second branchial arch
(E) Second branchial groove

A

The correct response is Option D.

Development of the six branchial arches occurs within the walls of the anterior foregut during the fourth week of gestation, as neural crest cells migrate into the future head and neck region and alternating ridges and depressions develop. A series of clefts forms during embryologic development to create the branchial grooves externally and the pharyngeal pouches internally.

The auricle arises from the first (mandibular) and second (hyoid) branchial arches and is further defined by the development of six hillocks, which appear on these arches during the sixth week of gestation. The anterior (first through third) hillocks give rise to the tragus, root of the helix, and superior helix. The posterior (fourth through sixth) hillocks give rise to the posterior helix, antihelix, antitragus, and lobule.

Meckel’s cartilage is derived from the first branchial arch; it ossifies to form the malleus and incus. The external acoustic meatus develops from the first branchial groove. The middle ear and eustachian tube are formed from the first pharyngeal pouch. Reichert’s cartilage is also derived from the second branchial arch and ossifies to form the stapes. The second, third, and fourth branchial grooves are obliterated within the cervical sinus during the later stages of development.

67
Q

Stahl’s ear deformity is characterized by which of the following physical findings?

(A) Absence of the antihelical fold
(B) Adherence of the ear to temporal skin
(C) Downward folding of the superior helix
(D) Presence of a third crus
(E) Prominence of the superior and lower poles

A

The correct response is Option D.

Stahl’s ear deformity is characterized by an abnormal third crus that extends from the antihelix to the helix. In this deformity, the usual superior crus is absent, and the scaphoid fossa is broad and flat. This gives the ear a pointed shape and accounts for its nicknames, such as satyr ear, Spock’s ear, Vulcan ear, or baboon ear.

Prominent ears are associated with the absence of an antihelical fold. Cryptotia is the adherence of the ear to the temporal skin. Lop ear is a downward fold of the superior helix. A telephone deformity, seen after otoplasty, is prominence of the upper and lower poles.

Numerous ear deformities may be seen, especially in newborns. Correct diagnosis is necessary for treatment planning and informing the child’s family about the abnormality. Stahl’s ear deformity results from a development error around the third month of gestation. Theories about its etiology include abnormal development of musculature, abnormal perichondrial growth, and failure of programmed recession. This deformity was originally described by Stahl in the 19th century. It is usually unilateral and occurs most commonly in Asians.

In newborns, Stahl’s ear deformity can be treated by molding the ear with splints or soft compounds, usually over the first six weeks of life. Success rates of 80% are reported, although controlled studies are lacking. Various surgical options are available for older children or adults. Usually, the abnormal crus is resected and may be used as graft material for the superior crus. Other surgical options rely on sutures alone or Z-plasty techniques. The technique used must be individualized to the specific deformity and the degree of severity.

68
Q

A 45-year-old woman comes to the office for consultation regarding reconstruction of the left ear six months after she sustained injuries during a motor vehicle collision. Injuries included avulsion of the entire external ear, except for the tragus, and avulsion of a 15 - 20-cm area of skin and fascia from the postauricular and temporoparietal region. Initial closure of the wounds was accomplished with skin grafting. Which of the following reconstructive techniques will result in the best aesthetic appearance of the ear?

(A) Placement of a porous polyethylene implant and coverage
(B) Placement of a porous polyethylene implant and coverage with a free radial forearm flap
(C) Placement of osteointegrated screws and a prosthetic device
(D) Rib cartilage grafting and coverage
(E) Rib cartilage grafting and coverage with a free radial forearm flap

A

The correct response is Option C.

A prosthetic device offers the best aesthetic appearance in this patient because she has no local skin available for framework coverage.

For good results from total auricular reconstruction, rib cartilage grafting generally requires coverage with healthy skin from the postauricular area. Temporoparietal fascial flaps are an alternative covering for cartilage grafts but usually are reserved for problems with graft exposure. Excellent results with minimal complications have been reported with porous polyethylene implants covered with temporoparietal fascial flaps for auricular reconstruction in burn patients. Previous reports on the use of polymeric silicone (Silastic) implants have shown unacceptably high rates of complications that resulted in implant removal. With any alloplastic material, adequate soft-tissue coverage is essential for a good outcome.

In this patient, the temporoparietal flap and local skin are unavailable, so a free tissue transfer would be needed for adequate coverage of an implant or graft. The radial forearm flap is the thinnest, most pliable free flap and offers reasonable coverage results, but provides a poor color match and suboptimal definition. For this patient, the reconstructive technique that results in the best appearance of the ear is a prosthetic ear produced by a medical artist. Typically, these ears are realistic in appearance and can be secured by adhesive or an osteointegrated screw or magnet. The main problem with these prosthetic devices is patient noncompliance in wearing them and the potential for patient embarrassment if they become dislodged.

69
Q

The ear lobule is innervated by which of the following nerves?

(A) Auriculotemporal
(B) Great auricular
(C) Greater occipital
(D) Lesser occipital
(E) Vagus

A

The correct response is Option B.

The ear is innervated by multiple nerves. The greater auricular nerve is a branch of C2 and C3. It travels on the superficial surface of the sternocleidomastoid muscle and enters the lower, posterior surface of the ear. Its branches supply the lobule as well as the helix, antihelix, and most of the cranial surface of the ear.

The auriculotemporal nerve is a branch of the third division of the trigeminal nerve and enters the ear near the tragus. It supplies the tragus and the root of the helix. The greater occipital nerve, which is a branch of C2 and C3, supplies the posterior scalp. The lesser occipital nerve is also a branch of C2. It sends off an auricular branch that supplies the upper third of the cranial surface of the ear. The vagus (X) nerve supplies the concha, via its branch called Arnold’s nerve.

70
Q

A 58-year-old woman is scheduled to undergo bilateral upper eyelid blepharoplasty. Preoperatively, the surgeon plans to resect skin, a strip of orbicularis oculi muscle, and a small amount of orbital fat. During the dissection, there is concern that the levator tendon has been lacerated just above the tarsal plate. This can be confirmed by visualizing which of the following structures through the laceration?

(A) Capsulopalpebral fascia
(B) Müller’s muscle
(C) Orbital septum
(D) Retro-orbicularis oculi fat
(E) Whitnall ligament

A

The correct response is Option B.

Superior to the tarsus, the layers of the eyelid are the conjunctiva, Müller’s muscle, levator tendon, orbital fat, orbital septum, retro-orbicularis oculi fat, orbicularis oculi muscle, and skin. Because the Müller’s muscle lies just below the levator tendon, visualization of this muscle confirms laceration of the tendon.

The conjunctiva is located deep to the Müller’s muscle. In this procedure, the conjunctiva would not be visualized unless the Müller’s muscle had been lacerated also. The orbital septum and retro-orbicularis oculi are anterior and superior to the levator tendon. The Whitnall ligament also is superior to the levator tendon.

71
Q

A 6-year-old girl with prominent ears due to overdeveloped conchae and effaced antihelical folds is scheduled to undergo otoplasty by a combination of methods including Stenstrom cartilage abrasion. In this technique, which of the following surfaces of the auricular cartilage is abraded?

(A) Anterior surface of the antihelix
(B) Anterior surface of the conchal bowl
(C) Lateral border of the helix
(D) Posterior surface of the antihelix
(E) Posterior surface of the conchal bowl

A

The correct response is Option A.

Classic congenital prominence of the ears is caused by an overdeveloped or excessively deep conchal bowl; an underdeveloped, effaced, or absent antihelical fold; or both. Surgical correction of prominent ears commonly calls for a combination of procedures, including the Mustardé technique for placement of mattress sutures to create an antihelical fold, partial excision of the conchal bowl, placement of concha-mastoid sutures for setback of the conchal bowl, and Stenstrom cartilage abrasion.

Stenstrom cartilage abrasion requires partial thickness scoring, scratching, or abrading of the anterior surface of the antihelix. This causes the cartilage to bend away from the abraded surface, creating or accentuating an antihelical fold.

To correct prominent ears, projection of the conchal bowl usually needs to be reduced by excision or by setback with placement of concha-mastoid sutures, not by scoring the anterior or posterior surface. Because the helix is normal in prominent ears, it should not be abraded.

72
Q

Which of the following surgical techniques for correction of prominent ears relies on placement of conchomastoid sutures?

(A) Furnas
(B) Luckett
(C) Mustardé
(D) Stenstrom
(E) Webster

A

The correct response is Option A.

The Furnas technique is the only method known for placement of conchomastoid sutures. It is designed to correct ears with deep conchae (more than 2.5 cm). The Mustardé technique recreates the antihelical fold with a series of molding sutures on the posterior aspect of the ear cartilage. The Stenstrom technique applies the Gibson and Davis principle, which states that cartilage tends to curl away from a cut surface. With this technique, the anterior scaphal surface is scored through a small medial incision near the cauda helicis to produce an antihelical fold. The Webster technique corrects a prominent helical tail by fixation of the helical tail to the concha. A Luckett technique relies on the excision of postauricular skin to achieve ear setback.

73
Q

Which of the following hormones is responsible for the success of molding therapy of ear deformities in the neonatal period?

(A) Estrogen
(B) Growth hormone
(C) Progesterone
(D) Prolactin
(E) Thyroid

A

The correct response is Option A.

At birth, the neonate’s ears are soft and pliable, which makes molding therapy of ear deformities more successful during the neonatal period. This pliability is thought to be due to the effect of maternal estrogen, which increases the amount of hyaluronic acid, a major constitute of ear cartilage. After six weeks, the infant’s estrogen level begins to fall, and the ears becomes firmer and less malleable. Because breast-fed infants have higher levels of estrogen for a longer time, their ears may be moldable for a longer period.

The other hormones have no effect on the pliability of cartilage in the newborn period.

Several studies reported success with different molding methods. One reported an 87% rate of success after treating 92 ears with soft putty and taping. The deformities treated included lop, prominent, Stahl’s, and constricted ears. Another study used flexible plastic tubing and wire to form splints and reported good results in 19 neonates. Children treated after three months of age showed little significant improvement. In both studies, complications were minimal.

74
Q

Which of the following is the principal disadvantage of banking amputated ear cartilage in a subcutaneous location for future use?

(A) Calcification
(B) Infection
(C) Pain
(D) Vascular ingrowth
(E) Warping

A

The correct response is Option E.

The cartilage of the ear is a delicate and unique structure of rolls and folds that is not easily reconstructed. For this reason, various methods for saving large pieces of amputated cartilage have been described. Some authors advocate banking the cartilage in a subcutaneous pocket in the abdomen. However, later retrieval of this cartilage often yields cartilage that has lost its strength and architectural detail. Other authors advocate burying the dermabraded cartilage in a postauricular flap or under a radial forearm flap and then transferring the flap to the head as a free flap. Although all of these methods may be successful, warping and contraction of avascular cartilage commonly cause loss of detail. Fibrosis of scar tissue and covering of the cartilage by overly thick tissue also obscure detail. Because of these disadvantages, microvascular replantation should be used, when possible, to give the best chance for preserving the native auricular architecture.

Banked cartilage can have some areas of calcification, but this does not cause significant problems. Infection is possible with banked cartilage. However, banking is usually attempted only if the cartilage is relatively clean, which greatly reduces the risk of infection. Pain is minimal in the area where the cartilage is implanted. Vascular ingrowth is not a disadvantage; it allows the cartilage to take but is easily divided when the cartilage is to be transferred.

75
Q

A 7-year-old boy with severe microtia is brought to the office by his parents for consultation regarding total reconstruction of the ear. The parents ask whether synthetic material could be used as the graft material instead of creating an additional scar on the child’s body. The boy=s own cartilage is preferable because of the decreased risk of which of the following complications?

(A) Extrusion
(B) Hematoma
(C) Hypertrophic scarring
(D) Pneumothorax
(E) Warping

A

The correct response is Option A.

Most experience with alloplastic materials in reconstruction of the ear has been with silicone. The other material that has been used commonly is porous polyethylene. The thin soft-tissue cover predisposes extrusion with alloplastic materials. According to a recent review article, in cases in which silicone and porous polyethylene were used, the following incidences were noted: infection, 9.4%; exposure/extrusion, 17%; and inflammatory reaction, 0.6%. Warping and distortion of autologous cartilage are troublesome and have been reported to occur in up to two thirds of cases. Incidence of hematoma and scarring is comparable with use of silicone and porous polyethylene, whereas the incidence of pneumothorax and warping is greater with autologous cartilage due to the nature of the procedure and cartilage itself.

76
Q

For each deformity of the ear, select the corresponding anatomic site in the photograph shown on page 44 (A-E).

(A) Concha cavum
(B) Helical rim
(C) Scapha
(D) Superior crus of the antihelix
(E) Triangular fossa

1 Effacement of this structure most commonly results in prominence of the ear.

2 Hypertrophy of this structure most commonly results in prominence of the middle third of the ear.

A

The correct response for Item 1 is Option D and for Item 2 is Option A.

In children with congenital ear prominence, the superior and middle thirds of the ear are most likely to be affected. The most likely cause of a prominent superior third of the ear is absence or effacement of the superior crus of the antihelix. As a result, the conchoscaphal angle is greater than 90 degrees and the helix is positioned more than 12 to 15 mm from the temporal region. The cephaloauricular angle is also increased, typically measuring more than 25 degrees. Appropriate management involves scoring and suturing of the cartilage to recreate the natural roll of the antihelix.

In contrast, prominence of the middle third of the ear is most likely caused by hypertrophy of the concha cavum. In affected patients, the concha cavum has a depth of more than 1.5 cm. The middle third of the ear is located more than 16 to 18 mm from the mastoid region. Options for correction include excision and/or reduction of the concha or setback with concha-mastoid sutures.

The scapha is the concave region between the helical rim and antihelix, and the triangular fossa is the concave area between the superior and inferior crura of the antihelix. Although the helical rim, scapha, and triangular fossa lie in the superior third of the ear, they do not typically cause prominent ears.

77
Q

A 77-year-old man has a 12-mm squamous cell carcinoma on the lateral margin of the right helix. He is scheduled to undergo excision of the lesion with confirmation of margins by frozen section, followed by immediate reconstruction. Which of the following flaps is most appropriate for ear reconstruction?

(A) Antia-Buch flap
(B) Postauricular flap
(C) Temporoparietal fascial flap
(D) Temporalis muscle flap

A

The correct response is Option A.

The Antia-Buch flap is most appropriate for reconstruction of this patient’s ear defect. The lesion can be excised easily because of its location on the lateral rim and of the size of the auricle. Following excision, the resultant defect is effectively reconstructed using the Antia-Buch flap, which is a local flap that uses tissue from the helical rim based on the postauricular skin to reconstruct the helical margin. It is a reliable, single-stage procedure that is acceptable aesthetically. The surgeon may need to excise a “dog ear”-shaped area of tissue from the conchal bowl and incise and advance the helical margins separately. However, because the two ears are not viewed simultaneously, moderate differences in ear size are frequently unnoticed.

A postauricular flap does not provide thin, contoured, helical-type tissue and requires several procedures for adequate coverage. The temporoparietal fascial flap provides thin, pliable soft-tissue coverage for a cartilage or alloplastic framework, as in patients undergoing microtia reconstruction. A temporalis muscle flap is excessively bulky and is not appropriate for ear reconstruction because it would obliterate the intricate detailing of the ear.

78
Q

A 14-year-old boy sustains an avulsion injury involving the entire pinna when he is bitten by a dog. The amputated part has been preserved on iced saline gauze. Following administration of antibiotics, tetanus toxoid, and rabies prophylaxis, microsurgical replantation of the ear is to be performed. Anastomosis of the arteries is most appropriate at which of the following anatomic locations on the ear?

(A) Anterior surface
(B) Inferior surface
(C) Posterior surface
(D) Superior surface

A

The correct response is Option C.

In patients undergoing microsurgical replantation of the ear, the tissues are debrided first, and dissection is performed to locate the appropriate vessels for replantation, with visualization provided by an operating microscope. Because the large arteries to the ear enter on the posterior aspect of the pinna, anastomosis is most appropriate on the posterior surface. These arteries include branches of the external carotid artery, the anterior auricular branch of the superficial temporal artery, and a branch of the occipital artery. In contrast, the smaller branches are located on the anterior surface.

79
Q

A 15-year-old boy undergoes reconstruction of a 15-mm2 traumatic defect of the right ear with a graft harvested from the contralateral ear. On examination two days after the procedure, the graft appears dusky. A photograph is shown above. Which of the following is the most appropriate next step in management?

(A) Hyperbaric oxygen therapy
(B) Application of leeches
(C) Release of the sutures
(D) Debridement of the graft

A

The correct response is Option A.

In this 15-year-old boy who exhibits duskiness at the graft site two days after undergoing composite grafting of the ear, the most appropriate next step is initiation of hyperbaric oxygen therapy. This will provide oxygenation during the critical ischemia period for the graft and thus is likely to improve the outcome. Hyperbaric oxygen therapy enhances antimicrobial activity by facilitating the oxidative burst of polymorphonuclear neutrophils. It increases the hyperoxygenation of tissue to a level that is 10 to 15 times greater than normal. In addition, it stimulates angiogenesis and blunts the ischemia-reperfusion injury response.

Application of leeches is appropriate if arterial input is adequate but venous outflow is insufficient, as in patients undergoing microsurgical replantation who demonstrate thrombosis of the vein, or if a suitable vein does not exist for anastomosis. However, duskiness of the ear is an indication of arterial insufficiency, and leeches would fail to attach if they were applied.

Because a composite graft receives its vascularity through diffusion from the surrounding wound bed, releasing the sutures would inhibit the “take” of the graft to the bed. Similarly, performing debridement two days after grafting is excessive. Instead, the composite graft should be left in place for a minimum of two weeks in order to demonstrate healing and incorporation, as long as infection does not develop.

80
Q

A 24-year-old man has pain and swelling of the left ear after injuring the ear in a fight. Physical examination shows obliteration of the normal contours of the lateral surface of the ear. Which of the following is the most appropriate management?

(A) Application of a pressure dressing for several days, followed by evacuation of clotted blood
(B) Needle aspiration of the ear
(C) Needle aspiration of the ear and application of a pressure dressing
(D) Incision and drainage of the skin and perichondrium and application of a pressure dressing
(E) Excision of thickened tissue and placement of suction-drainage catheters

A

The correct response is Option D.

Hematoma formation is the primary complication of blunt trauma to the ear. The mechanism of injury involves disruption of blood vessels in the perichondrium, leading to hemorrhage. The blood fills the space between the perichondrium and cartilage, distorting the contour of the lateral ear into a convex shape and blocking the vascular supply to the cartilage, which is derived from the perichondrium. Necrosis or infection of the cartilage results.

Prompt treatment involves removing the accumulated blood while maintaining pressure on the affected area for several days to prevent recurrence. To accomplish this, an incision is made through the skin and perichondrium on the inner side of and parallel to the antihelix, which will conceal the scar. The blood is drained and the wound is inspected for further bleeding. When the surgeon is assured that the bleeding has stopped, a pressure dressing and a head dressing are applied.

Late treatment of a cauliflower ear deformity involves excision of the thickened tissue, including fibrous tissue and new cartilage, followed by application of a pressure dressing.

Simple needle aspiration of the blood is likely to result in development of seroma.

81
Q

A 21-year-old man sustains a complete amputation of the right ear at the level of the external auditory canal in a motor vehicle collision. There are no other injuries. Which of the following procedures will provide the best aesthetic result?

(A) Delayed total ear reconstruction with a rib cartilage graft
(B) Dermabrasion of the epidermis of the amputated ear, burial of the ear in a subcutaneous postauricular pocket, followed by removal and coverage with a skin graft or flap
(C) Removal of the skin of the amputated ear, reattachment of the ear cartilage, and immediate coverage with a temporoparietal fascial flap and skin graft
(D) Composite grafting of the amputated ear followed by surface cooling
(E) Microsurgical ear replantation

A

The correct response is Option E.

Successful microsurgical replantation of the ear provides superior aesthetic results while eliminating the need for other complex reconstructive procedures. However, this technique is associated with increased operative time and the need for multiple blood transfusions. Hospitalization is typically prolonged, and failure rates associated with the procedure are high.

Delayed reconstruction results in only moderate cosmetic improvement, and secondary reconstruction does not sufficiently recreate the intricate architecture of the external ear.

Primary nonvascularized replantation of the ear produces a good appearance initially because of the survival of the avulsed cartilage; however, late distortion of the cartilage frequently limits the overall aesthetic result. Techniques used for nonvascularized replantation include primary reattachment of the ear with surface cooling, dermabrasion of the ear, and partial or complete burial of the ear in a postauricular skin pocket, followed by coverage of the filleted cartilage with a temporoparietal fascial flap and skin graft.

82
Q

A 6-year-old boy has prominent ears. Physical examination shows an obtuse concha-mastoid angle. The antihelical fold is normal. Which of the following is the most appropriate management?

(A) Use of a headband splint at night
(B) Use of Mustardé sutures
(C) Setback of the concha using concha-mastoid sutures
(D) Excision of excess skin
(E) Resection of the concha

A

The correct response is Option C.

Because prominent ears can be caused by an enlarged conchal bowl, an obtuse concha-mastoid angle, or loss of the antihelical fold, appropriate management should be based on the cause of the deformity. This 6-year-old boy has ear prominence caused by an obtuse concha-mastoid angle. Conchal setback is recommended in children to correct the enlarged conchal bowl and obtuse concha-mastoid angle deformities. This is accomplished using concha-mastoid sutures, which are mattress sutures placed between the posterior conchal wall and the mastoid periosteum to create a more acute concha-mastoid angle and decrease the height of the protruding concha.

In contrast, elliptical conchal excision may be required to perform conchal setback in adults, whose ear cartilage is typically stiff.

Ear prominence resulting from loss of the antihelical fold is best corrected by abrading or scoring the antihelix and placing Mustardé mattress sutures between the conchal eminence and the scaphoid eminence.

Splinting is effective only in infants because of the pliability of the ear cartilage in this age group.

Excess skin may need to be excised following placement of concha-mastoid sutures, but this is unlikely to produce adequate setback if performed alone. This technique is appropriate instead to decrease the prominence of the lobule or superior helix.

83
Q

Each of the following deformities is commonly associated with microtia EXCEPT

(A) cervical spine abnormalities
(B) inner ear abnormalities
(C) macrostomia
(D) mandibular hypoplasia
(E) preauricular pits

A

The correct response is Option B.

Patients with microtia have partial or complete absence of the external ear structures due to abnormal embryologic development of portions of the first, or mandibular, and second, or hyoid, branchial arches. This typically occurs during the fourth to twelfth week of intrauterine development and affects the auditory ossicles, external auditory canal, middle ear cavity, and tympanic membrane. Several abnormalities can occur in conjunction with microtia. Orbital auricular vertebral syndrome, also known as Goldenhar syndrome, and the Tessier No. 7 cleft also result from abnormalities in the development of the first and second branchial arches. Orbital auricular vertebral syndrome is characterized by microtia, cervical spine abnormalities, mandibular hypoplasia, preauricular pits and sinuses, and hemifacial microsomia. The Tessier No. 7 cleft manifests as microtia, macrostomia, and preauricular sinuses.

Because the external auditory meatus and internal ear are derived from different structures, the internal ear is usually well constructed in patients with microtia. Likewise, patients with orbital auricular vertebral syndrome have abnormalities of the middle and external ear but not the inner ear.

84
Q

A 7-year-old boy has cryptotia. Which of the following operative techniques is most appropriate for correction?

(A) Advancement of the third crus of the antihelix
(B) Partial detachment of the folded segment of helical cartilage from the scapha and repositioning of the helix with sutures
(C) Placement of sutures from the conchal bowl to the mastoid fascia
(D) Rasping of the anterior surface of the antihelical cartilage to create the antihelix
(E) Separation of the superior auricle from the temporal skin with placement of a retroauricular skin graft

A

The correct response is Option E.

In patients with cryptotia, the superior portion of the auricle adheres to the temporal skin in varying degrees of severity. Management involves release of the adherent portion of the auricle; skin grafting is frequently required. Advancement of the third crus of the antihelix is the treatment of Stahl’s ear, a deformity that involves the presence of a third antihelical crus.

Partial detachment of the helix from the scapha and resuturing of the helix at a more appropriate angle are recommended for patients who have the constricted ear deformity. In this deformity, which varies in severity, the helical rim is constricted, and the superior portion of the helix typically folds over the scapha.

Placement of sutures from the conchal bowl to the mastoid fascia and rasping of the anterior surface of the antihelical cartilage have been described for correction of prominent ears, which are characterized by widening of the conchoscaphal angle and flattening of the antihelical fold. Other methods of correction include resection of the conchal bowl and placement of retention sutures, which are used to recreate the antihelical fold.

85
Q

A 24-year-old man undergoes total auricular reconstruction using a porous polyethylene implant covered with a temporoparietal fascia flap and a skin graft after sustaining burns to the face and ear. On follow-up examination one month later, there is a 1 * 1-cm area of necrosis of the flap; a small area of implant is exposed beneath the eschar.

Which of the following is the most appropriate next step in management?

(A) Initiation of frequent dressing changes and allowing the wound to heal secondarily
(B) Removal of the implant and allowing the wound to heal secondarily
(C) Removal of the implant and immediate replacement with a smaller porous polyethylene implant
(D) Removal of the implant and immediate replacement with a rib cartilage graft
(E) Removal of the implant and immediate replacement with a Silastic implant

A

The correct response is Option A.

Excellent results have been reported with the use of porous polyethylene implants covered with temporoparietal fascial flaps for ear reconstruction in patients with burns. In the past, Silastic implantation was associated with an unacceptably high complication rate, frequently resulting in removal of the implant. Adequate soft-tissue coverage of any type of alloplastic implant, whether porous polyethylene or Silastic, is essential.

Because porous polyethylene allows for ingrowth of tissue, small areas of implant exposure can be managed with frequent dressing changes and either allowing the wound to heal secondarily or covering the wound with a skin graft after granulation tissue has formed over the implant. Although removal is an option, the surgeon should wait for a minimum of six months before inserting another implant or a cartilage graft.

86
Q

Which of the following best describes the Mustarde otoplasty technique?

(A) Bending of the antihelix using mattress sutures
(B) Endoscopic scoring and suturing of the cartilage
(C) Excision of a crescent-shaped piece of skin and cartilage
(D) Placement of sutures from the concha to the mastoid
(E) Scoring of the anterior surface of the antihelix

A

The correct response is Option A.

The Mustarde technique, which was first described in 1963, involves the placement of permanent sutures through the cartilage and perichondrium on the cranial portion of the ear to bend the antihelix posteriorly. Although this technique is still preferred, many surgeons use a combination of procedures based on the deformity and the needs of each patient.

Stenstrom’s technique is used to correct prominent ears. With this technique, the anterior surface of the antihelix is bent and scored to create a posterior roll. This can be accomplished using an otoabrader instrument.

Graham and Gault conceived an endoscopic technique that involves scoring and suturing of the posterior cartilage. Scars are minimized with this procedure.

With the Luckett procedure, a crescent-shaped portion of skin and cartilage is excised from the length of the antihelix. The edges of the cartilage are sutured to create an antihelical fold.

In the Furnas technique, sutures are placed from the concha to the mastoid to diminish the size of the concha. Excision of conchal cartilage may also be required.

87
Q

In a child with microtia, growth of the reconstructed ear is primarily dependent on which of the following factors?

(A) Age of the child
(B) Presence of the perichondrium
(C) Preservation of intercostal muscle
(D) Type of fixation
(E) Type of rib graft

A

The correct response is Option B.

An intact perichondrium is most crucial for growth of the reconstructed ear. If the perichondrium is present, the reconstructed auricular framework will grow at a rate similar to that of the normal ear. The age of the child and timing of reconstruction are secondary considerations because the auricle reaches near normal size at approximately 6 years of age. Preservation of the intercostal muscle attached to the graft does not influence future growth of the reconstructed ear, nor does the type of fixation used. Only rib cartilage should be used for grafting, and bone should not be included.

88
Q

Which of the following congenital ear deformities is characterized by absence of the superior auriculocephalic sulcus?

(A) Cryptotia
(B) Cup ear deformity
(C) Lop ear
(D) Microtia
(E) Prominent ear deformity

A

The correct response is Option A.

Cryptotia (“hidden ear”) is a congenital deformity of the cartilage of the scapha and antihelix. In neonates who have this deformity, the upper pole of the ear is buried beneath the scalp, and the superior auriculocephalic sulcus is absent. Conservative management is most appropriate initially; surgical release should be performed when the child is older.

Infants with the cup ear deformity have hooding of the scapha and helix and flattening of the antihelix. Lop ear is characterized by protrusion of the ear and folding of the superior helix. Microtia is a hypoplastic condition that manifests as varying degrees of ear absence, from anotia (complete ear absence) to a smaller than normal ear with normal morphology. The prominent ear deformity involves widening of the conchoscaphal angle, increased auriculocephalic distance, and loss of the antihelical fold.

89
Q

Which of the following congenital ear deformities is characterized by absence of the superior auriculocephalic sulcus?

(A) Cryptotia
(B) Cup ear deformity
(C) Lop ear
(D) Microtia
(E) Prominent ear deformity

A

The correct response is Option A.

Cryptotia (“hidden ear”) is a congenital deformity of the cartilage of the scapha and antihelix. In neonates who have this deformity, the upper pole of the ear is buried beneath the scalp, and the superior auriculocephalic sulcus is absent. Conservative management is most appropriate initially; surgical release should be performed when the child is older.

Infants with the cup ear deformity have hooding of the scapha and helix and flattening of the antihelix. Lop ear is characterized by protrusion of the ear and folding of the superior helix. Microtia is a hypoplastic condition that manifests as varying degrees of ear absence, from anotia (complete ear absence) to a smaller than normal ear with normal morphology. The prominent ear deformity involves widening of the conchoscaphal angle, increased auriculocephalic distance, and loss of the antihelical fold.

90
Q

A 25-year-old woman has facial asymmetry. She says that she has had progressive loss of soft-tissue volume on the right side of the face since age 10 years that became stabilized four years ago. Examination shows significant subcutaneous atrophy of the right side of the face without bony asymmetry. She also has hypopigmentation of the iris on the affected side.

Which of the following is the most appropriate management?

(A) Bone graft augmentation of the midface
(B) Alloplastic augmentation
(C) Reconstruction with a microvascular serratus anterior free flap
(D) Reconstruction with a microvascular parascapular free flap
(E) Reconstruction with a superficial temporal fascia flap

A

The correct response is Option D.

This 25-year-old woman has Romberg’s hemifacial atrophy characterized by progressive unilateral loss of facial soft tissue. The underlying skeleton is also affected in patients with severe forms of the disease. Surgery should be delayed until the condition becomes stabilized, which is indicated by the cessation of facial atrophy. When this has occurred, a microvascular parascapular flap can be deepithelialized and customized to fit the dimensions of the defect, and then transferred and buried subcutaneously.

Skeletal augmentation with either bone graft or alloplast is not appropriate because the bones of the face are not affected. The serratus anterior flap would only atrophy over time, and the superficial temporal fascia flap would not provide the necessary volume.

91
Q

A 35-year-old man has persistent enophthalmos 18 months after undergoing open reduction of a fracture of the orbital floor and zygoma. Forced duction testing shows no restriction of eye motion. Which of the following is the most likely cause of this patient’s enophthalmos?

(A) Fat atrophy
(B) Fibrosis of the extraocular muscles
(C) Herniated contents of the orbit within the maxillary sinus
(D) Inadequate fracture reduction
(E) Scar contracture

A

The correct response is Option D.

Persistent enophthalmos following facial trauma is primarily caused by increased bony orbital volume secondary to inadequate fracture reduction. An appropriate anatomic reduction should be the initial goal of surgery; bone grafting may be required to restore orbital volume to normal levels.

Fat atrophy, extraocular muscle fibrosis, herniation of orbital contents, and scar contracture can also contribute to the discrepancy between traumatic and nontraumatic orbital volume, but are less likely to contribute to postoperative enophthalmos than inadequate fracture reduction. In addition, extraocular muscle fibrosis and/or scarring would typically be associated with limited motion of the globe on forced duction testing.

92
Q

A 7-year-old girl has conchal valgus and underfolding of the antihelix. The cranioauricular angle is 45 degrees. Which of the following is most appropriate for correction of this patient’s deformity?

(A) Conchoscaphoid suturing
(B) Customized splinting
(C) Flag flap transfer
(D) Helical release with skin grafting
(E) Posterior scoring of the antihelical cartilage

A

The correct response is Option A.

Prominent ear deformities affect 5% of the general population and are characterized by conchal valgus with a cranioauricular angle greater than 40 degrees and underfolding of the antihelix. Conchal hypertrophy is a rare finding. Several options exist for management of prominent ears during childhood, including conchoscaphoid (MustardŽ) or conchomastoid suturing, anterior scoring of the cartilage at the antihelical fold, resection of the concha, and resection of postauricular skin.

Although a customized splint can be applied for management of minor deformities of the auricular cartilage, including prominent, lop, cup, and Stahl’s ears, it is only effective during the neonatal period unless the deformity is associated with hypoplasia.

Flag flap transfer is useful for correction of a lop ear deformity.

Helical release with split-thickness skin grafting is appropriate management for cryptotia. In this technique, the superior portion of the auricle is buried beneath the skin.

As mentioned above, anterior, not posterior, scoring of the antihelical cartilage is a recognized technique for management of prominent ears.

93
Q

Which of the following is the most appropriate management of a 1-year-old boy who has isolated microtia on the left?

(A) Fabrication of a costal cartilage framework at age 3 years
(B) Implantation of a Silastic framework at age 4 years
(C) Placement of a bone-conduction hearing aid at age 5 years
(D) Creation of an ipsilateral ear canal at age 6 years
(E) Autologous ear reconstruction at age 7 years

A

The correct response is Option E.

Although the recommended age of surgery may vary among individual patients with microtia, autologous ear reconstruction is typically performed once the child reaches 5 years of age; most children are between the ages of 6 and 7 years at the time of surgery, at which time there is typically sufficient rib cartilage for reconstruction. The ear attains 85% of its total growth by age 3 years and has almost fully developed by ages 5 to 7 years; however, minimal changes in the width of the ear and its distance from the scalp can be seen until age 10 years.

Hearing in the affected ear should be assessed as soon as possible. When necessary, a bone-conduction hearing aid, whether external or implantable, should be initially used by age 1 year.

Implantation of a Silastic framework is not the first choice for management of congenital microtia. Creation of an ear canal is typically performed for unilateral microtia when the patient is age 13 to 19 years and should not be initiated until reconstruction of the external auricle has been completed.

94
Q

A 20-year-old man has severe right ear pain 24 hours after undergoing bilateral otoplasty. Which of the following is the most likely cause?

(A) Chondritis
(B) Excessively tight dressings
(C) Hematoma
(D) Nerve injury
(E) Otitis externa

A

The correct response is Option C.

Severe unilateral pain in the ear occurring within the first 24 hours after surgery is most consistent with the development of a hematoma resulting from increased pressure on the surrounding soft tissue. If the hematoma is not evacuated urgently, more severe complications, such as pressure necrosis of the overlying skin or underlying cartilage and fibrosis of the soft tissues, may result.

Chondritis, an uncommon complication of otoplasty, is characterized by pain, swelling, and tenderness of the affected ear. This condition initially presents several days after surgery and is best managed with intravenous antibiotic therapy. Surgical exploration is indicated in those patients diagnosed with suppurative chondritis.

Excessively tight dressings and head wraps would be detected soon after surgery, as the effects of the anesthesia dissipate, and would be associated with bilateral pain.

Patients who sustain injuries to the great auricular nerve during otoplasty would experience paresthesia and dysesthesia of the involved ear several weeks following surgery.

Otitis externa would not develop as soon as 24 hours after surgery.

95
Q

A 2-week-old neonate has bilateral prominent ears with lopped superior poles. Which of the following is the most appropriate management?

(A) Observation
(B) Molding the ears using tape and splinting
(C) Injection of a corticosteroid
(D) Otoplasty at age 2 years
(E) Otoplasty at age 6 years

A

The correct response is Option B.

In this neonate with bilateral prominent ears, immediate management should include molding of the ears with tape and splinting. Circulating maternal estrogens are still present until the age of 6 months, allowing for successful molding of the soft, malleable ear cartilage in infants with cryptotia and prominent ears. Molding can also be performed in newborns with lop ear and Stahl’s ear during the neonatal period. Complete correction without surgery is a realistic expectation.

Because the concept of body image typically begins to form at about school age, ear reconstruction, if required, should ideally be performed between the ages of 5 and 6 years. The normal ear is within 6 to 7 mm of its full vertical height by the age of 6 years, allowing for the construction of an ear that is symmetrical to the normal ear.

Observation alone is inadequate in a child with ear deformities, and injection of a corticosteroid is inappropriate management. Otoplasty should not be performed at the age of 2 years because ear growth is not complete.

96
Q

An 8-year-old boy has a third crus, flattening of the antihelix, and malformation of the scaphoid fossa. These findings are most consistent with which of the following?

(A) Constricted ear
(B) Cryptotia
(C) Prominent ear
(D) Stahl’s ear
(E) Telephone ear deformity

A

The correct response is Option D.

This child has findings consistent with Stahl’s ear, a rare congenital anomaly of unknown cause characterized by the presence of a third crus, flattening of the antihelix, and malformation of the scaphoid fossa. This condition is often difficult to correct surgically.

Constricted ear manifests as hooding of the helix and scapha. In patients with cryptotia, the upper pole of the ear cartilage is buried beneath the skin, and the superior auriculocephalic sulcus is absent. Prominent ears can be associated with several factors, including incomplete development of the antihelix, enlargement of the concha, widening of the conchoscaphal angle (ie, greater than 90 degrees), and protrusion of the ear lobe. The telephone ear deformity results from either excessive reduction of the concha or inadequate correction of prominent upper and lower poles of the ear during an otoplasty procedure.

97
Q

Which of the following structures provides sensation to the upper cranial surface of the ear?

(A) Anterior branch of the great auricular nerve
(B) Arnold’s branch of the vagus nerve
(C) Auriculotemporal nerve
(D) Lesser occipital nerve
(E) Posterior branch of the great auricular nerve

A

The correct response is Option D.

Branches of the lesser occipital nerve supply sensation to the upper cranial surface of the ear and skin of the anterior and superior surfaces of the external auditory canal. The anterior branch of the great auricular nerve (which forms from branches of cervical nerve roots C2-3 within the cervical plexus) supplies sensation to the lower half of the lateral surface of the ear, while the posterior branch innervates the lower portion of the cranial surface of the ear. The auriculotemporal nerve provides sensation to the anterosuperior surface of the external ear. Arnold’s nerve, which is a branch of the vagus nerve, supplies sensation to the skin of the concha and posterior ear canal.

98
Q

The most likely cause of isolated unilateral microtia in a neonate is maldevelopment of mesenchymal proliferations around which pharyngeal cleft(s) during which trimester?

A

The correct response is Option A.

The external ear develops during the end of the first trimester from six mesenchymal proliferations around the first pharyngeal cleft. These proliferations arise from the tissues of the first and second pharyngeal arches and are known as auricular hillocks. They slowly enlarge and eventually fuse to form the external ear. This process occurs between four and one half weeks and 10 weeks of intrauterine development as other facial structures are being formed.

Although the external ear, along with other facial structures, grows during the second trimester of fetal development, it has already been formed during the first trimester. Tissues around the second pharyngeal cleft contribute to the formation of the muscles of facial expression, the tonsils, the hyoid bone, and other structures but are not involved in the formation of the external ear.

99
Q

A 25-year-old man has complete loss of the upper two-thirds of the right ear two years after sustaining a burn injury to the ear. On examination, the ear lobe and lower part of the conchal cartilage are viable and have adequate skin coverage; the ear canal is open. Scarred skin surrounds the ear remnant.

Which of the following is the most appropriate operative procedure for correction of this patient’s deformity?

(A) Creation of the upper ear with a rib cartilage framework and coverage with a local skin flap
(B) Creation of the upper ear with a rib cartilage framework and coverage with a pre-expanded local skin flap
(C) Creation of the upper ear with a rib cartilage framework and coverage with a temporoparietal fascial flap and a split-thickness skin graft
(D) Creation of the upper ear with a Silastic framework and coverage with a local skin flap
(E) Creation of the upper ear with a Silastic framework and coverage with a temporoparietal fascial flap and a split-thickness skin graft

A

The correct response is Option C.

The most appropriate surgical procedure for correction of this patient’s deformity is creation of the upper ear using a rib cartilage graft and coverage with a temporoparietal fascia flap and a split-thickness skin graft. These procedures will most likely result in a satisfactory outcome for this difficult reconstructive problem. The rib cartilage can be carved into an appropriate framework and covered with a thin temporoparietal fascia flap; a thin split-thickness skin graft can be used to create the intricate detail of the external ear. When successful, this reconstruction will be durable and long-lasting.

Local skin is the coverage material of choice in classic microtia reconstruction; however, when the ear remnant is surrounded by scarred skin, as in this patient with a burn injury, it will not stretch adequately to cover the framework and show detail. Skin expansion will fail because scarred skin expands poorly.

Silastic frameworks can give good early results but are not long-lasting. Because even the most minor trauma or wound problem can lead to total loss of the reconstruction, Silastic frameworks are not a good choice for reconstruction.

100
Q

A 17-year-old high school wrestler had the sudden onset of swelling of the right ear 24 hours ago. Initial attempts at aspiration of the ear were unsuccessful. On current physical examination, there is a hematoma involving the upper half of the right ear.

Which of the following is the most appropriate management?

(A) Placement of an ice pack on the ear followed by observation
(B) Aspiration of the hematoma after liquefaction
(C) MRI of the ear for delineation of the cartilage fracture followed by surgical repair
(D) Surgical drainage of the hematoma followed by placement of through-and-through sutures with gauze bolsters
(E) Debridement of the skin over the hematoma followed by insertion of the ear cartilage into a postauricular skin flap

A

The correct response is Option D.

Separation of the skin of the external ear from the cartilage, and the subsequent development of a hematoma because of a shearing type injury, is a known injury in wrestlers and boxers. If not treated correctly it can result in a chronically scarred ear with no definition, commonly referred to as “cauliflower ear.”

Appropriate management of the acute hematoma in this patient is surgical drainage and placement of through-and-through sutures with gauze bolsters to coapt the separated skin to the underlying cartilage. These bolsters are left in place for seven to 10 days. Drainage and suturing can be accomplished under local anesthesia if the patient is cooperative; this procedure will typically allow the skin of the avulsed ear to adhere to the underlying cartilage and prevent the development of the deformity described previously.

Placement of an ice pack followed by observation would only delay appropriate treatment of this patient’s injury. Waiting for the hematoma to liquefy and then aspirating it will not prevent chronic scarring because the skin of the avulsed ear will not adhere correctly to the delicate cartilage framework. An MRI is not necessary and will only delay the appropriate treatment. Debridement of the viable ear skin would be contraindicated because it would only complicate the injury and its management.

101
Q

Six months after undergoing bilateral otoplasty for correction of prominent ears, a 27-year-old man has recurrent prominence of the upper half of the left ear. At follow-up examination one month after surgery, the ears appeared symmetric.

Which of the following is the most likely cause of this patient’s recurrent deformity?

(A) Disruption of the conchal-mastoid sutures
(B) Disruption of the sutures used to create the antihelical fold
(C) Inadequate excision of cartilage from the concha
(D) Inadequate excision of skin from the posterior ear
(E) Incorrect placement of the postoperative dressing

A

The correct response is Option B.

The most likely cause of this patient’s recurrent prominence of the upper half of the left ear is inadequate placement of sutures used in the creation of the antihelical fold. Lack of formation of the antihelical fold is one of the most common causes of prominent ears. Surgical creation of the fold can be accomplished with permanent sutures alone, cartilage incision or resection combined with sutures, or abrasion of the lateral ear cartilage with or without sutures. When used alone, sutures are usually very effective in the creation of the antihelical fold in children because their ear cartilage is soft and pliable. In adults, who have less pliable cartilage, sutures can be used alone to create a fold, but frequently other techniques must be performed to allow the cartilage to bend. When sutures are used alone, they must be permanent and precisely placed, or relapse can occur.

Excision of conchal cartilage, skin resection, placement of conchal-mastoid sutures, and placement of the postoperative dressing, while all vital aspects of otoplasty for prominent ears, are less likely to be involved in recurrence of the deformity than the inadequate creation of the antihelical fold, which is most commonly associated with recurrence of the deformity.